You are on page 1of 91

RESPIRATORY-System

Wise 1700-by Sush and Team. 2016


Susmita, Asad, Manu, Saima, Zohaib, Savia, Shanu, Mona, Manisha, Sitara, Samreena, Sami and Komal


Dear Plabber,


• This first ever System Wise 1700 document was created thanks to 3 months of daily

hard work by the PLAB Skype group ‘Unity’ which was brought together by Dr Susmita

Chowdhury.

• Please ignore the old versions posted by my new skype member Murtaza as he did so

without permission.


The team members were:


& Susmita (Lead/most ignorant as she is working full time in public health for 13 years)

& Asad (Invaluable in IT and all types of support/the heart of the group)

& Manu (Volunteered to solve more questions/pathologist/amazing genuine person)

& Saima (Most concise clear notes/ photographic memory)

& Zohaib (Great research/a surgeon)

& Savia (Great research/multi-tasker with two little ones)

& Shanu (Very helpful after her March exam for those appearing in June)

& Mona (Great contributor in discussions)

& Manisha (Gyne/great discussion contributor)

& Sitara (Good discussion contributor)

& Samreena (Stayed a shorter time but great)

& Sami (Contributed the most early on but too brilliant for the group/still great friends)
Komal (Knowledgeable sweet supportive girl)

&


• The main purpose was to break down the 1700 Q Bank System wise.


• We did our own reliable research for the options (OHCM/Patient info etc.) and

concluded these keys below on skype. This can save you 100s of hours of research. But I

suggest you also do your own.


• 90% of the document consists of Unity research. We also added information from other
circulating documents and they are referenced as Dr Khalid/Dr Rabia (and her Team).


• However, several keys may be ‘incorrect’ and so please use your own judgment as we

take no responsibility. I suggest cross checking with Dr Khalid’s latest keys (a few of

which are still debatable). Finally decide on your own key.


• Sorry if some members failed to make their answers thorough. The highlights are mostly

as per what the team members wanted to highlight. Blank tables to be ignored.


• Note that some 1700 Questions are missing from here (when members did not do their

share). Questions may not be in order due to merging of documents and there is excess

information than required. Read as much as needed.


• This has been circulated by our team as a generous contribution to the Plabbers’ success

and must not be ‘sold’.

Good luck and best wishes: Sush and Team
1
RESPIRATORY-System Wise 1700-by Sush and Team. 2016
Susmita, Asad, Manu, Saima, Zohaib, Savia, Shanu, Mona, Manisha, Sitara, Samreena, Sami and Komal

14. A 5yo with recurrent chest pain, finger clubbing with offensive stool. Choose the
single most likely investigation?
a. Endomysial/Alpha gliadin antibody
b. Sweat test
c. Barium meal
d. ECG
e. Glucose tolerance test

Q. 1. What is the likely key?
Q. 2. What is the diagnosis?
Q. 3. What is the mode of inheritance?

Ans. 1. B.
Ans. 2. Cystic fibrosis
Ans. 3. Autosomal recessive

SWEAT TEST – used to screen for CYSTIC FIBROSIS (OHCM PG166 9th Edition)

Cystic fibrosis is a genetic condition in which the lungs and digestive system become
clogged with thick sticky mucus due to defect in the Cl- channel ! defective chloride
secretion & increased sodium absorption across airway epithelium ! predispose
LUNG to ‘CHRONIC PULMONARY INFECTION & BRONCHIECTASIS.

*defect due to mutation in CFTR on chromosome 7 (cystic fibrosis transmembrane
conductance regulator gene)

Signs/Symptoms

Finger Clubbing
Cyanosis
Bilateral Coarse Crackling

(in neonate: failure to thrive, meconium ileus and chest infection)
(in children & young adults: cough, wheezes, recurrent infection and Bronchiectasis)

*between 18 – 30 age, patient develop ‘BRONCHIECTASIS’ (abnormal widening of
Bronchi or their branches) ---- keep in mind the age

Investigation

Sweat test: sweat sodium and chloride >60mmol/L; (chloride usually > sodium)

Sinus X ray or CT scan - Opacities of sinuses.

Lung function tests, LFTs, Sputum microbiology.


2
RESPIRATORY-System Wise 1700-by Sush and Team. 2016
Susmita, Asad, Manu, Saima, Zohaib, Savia, Shanu, Mona, Manisha, Sitara, Samreena, Sami and Komal

WHY NOT KEY A – Endomysium – layer of connective tissue covering muscle fibre. It
contains ‘Transglutaminase Tissue’ and antibodies that bind to this transglutaminse
are called ‘EmA’ and they are detected in ‘COELIAC DISEASE’. (OHCM PG 280 9th
Edition)

Celiac Disease is an autoimmune disorder of SMALL INTESTINE
Symptoms = STEATORHEA (pale and greasy loose stool) , failure to thrive (children),
fatigue, weight loss, Anemia (decreased IRON or B12) and Vit deficiency because
Small Intestine not absorbing nutrients .

“DIAGNOSIS: Cystic FIbrosis. Mutation of CFTR on chromosome 7.
It causes dehydration. Hence, bronchiectasis, bowel obstruction and bacterial
growth,

Signs
• Finger clubbing.
• Cough with purulent sputum.
• Crackles.
• Wheezes (mainly in the upper lobes).

INVS:

Sweat Test. Chloride concentration > 60. Sinus X ray or CT scan - Opacities of
sinuses.

Lung function tests, LFTs, Sputum microbiology.”

17. A man with suspected active TB wants to be treated at home. What should be
done to prevent
the spread of disease?
a. Immediate start of the tx with Anti-TB drugs
b. All family members should be immediately vaccinated with BCG vaccine
c. Patient should be isolated in a negative pressure chamber in his house
d. Universal prevention application protocol

Ans. D. Universal prevention application protocol.

This protocol isn’t available anywhere on the internet. Everybody is suggesting D
on the
basis of exclusion.

60. A 26yo man present to ED with increasing SOB on left side and chest pain. He has
been a heavy smoker for the past 4 years. He doesn’t have any past med hx. What is
the likely dx?
a. Pulmonary embolism
b. MI

3
RESPIRATORY-System Wise 1700-by Sush and Team. 2016
Susmita, Asad, Manu, Saima, Zohaib, Savia, Shanu, Mona, Manisha, Sitara, Samreena, Sami and Komal

c. Asthma
d. Pleural effusion
e. Pneumothorax
Q. 1. What is the key?
Q. 2. What are the points in favour of your diagnosis?
Q. 3. What is the cause of the disease in this case?



Ans. 1. The key is e. Pneumothorax.
Ans. 2. Increased shortness of breath and chest pain with no past medical history.
Ans. 3. Heavy smoking. Tobacco is a risk factor for spontaneous pneumothorax.


Pneumothorax (OHCM PG 182 & 824)

Causes: Often spontaneous (especially in young thin men – see age) due to rupture
of a sub pleural bulla (airspace = rupture = PNEUMOTHORAX

Other causes: asthma; COPD; TB; pneumonia; lung abscess; carcinoma; cystic
fibrosis; lung fibrosis; sarcoidosis; connective tissue disorders (Marfan’s sy., Ehlers–
Danlos sy.), trauma; iatrogenic (subclavian CVP line insertion, pleural aspiration/
biopsy, trans bronchial biopsy, liver biopsy, positive pressure ventilation).

Symptoms: There may be no symptoms (especially if fit, young and small
pneumothorax) or there may be sudden onset of dyspnea and/or pleuritic chest
pain. Patients with asthma or COPD may present with a sudden deterioration.
Mechanically ventilated patients may present with hypoxia or an increase in
ventilation pressures.

Signs: Reduced expansion, hyper-resonance to percussion and diminished breath
sounds on the affected side. With a tension pneumothorax, the trachea will be
deviated away from the affected side.

CXR – should not be performed if tension pneumothorax (pg 763) DECOMPRESSION
if TENSION PNEUMOTHORAX

This is medical emergency !

Essence! Air drawn into the pleural space with each inspiration has no route of
escape during expiration. The mediastinum is pushed over into the contralateral
hemithorax, kinking and compressing the great veins. Unless the air is rapidly re-
moved, cardiorespiratory arrest will occur.

Signs! Respiratory distress, tachycardia, hypotension, distended neck veins,


trachea deviated away from side of pneumothorax. Increased percussion note,
reduced air entry/breath sounds on the a ected side.

4
RESPIRATORY-System Wise 1700-by Sush and Team. 2016
Susmita, Asad, Manu, Saima, Zohaib, Savia, Shanu, Mona, Manisha, Sitara, Samreena, Sami and Komal

Treatment ! To remove the air, insert a large-bore (14–16G) needle with a syringe,
partially filled with 0.9% saline, into the 2nd intercostal interspace in the
midclavicular line on the side of the suspected pneumothorax. Remove plunger to
allow the trapped air to bubble through the syringe (with saline as a water seal) until
a chest tube can be placed. Alternatively, insert a large-bore Venflon in the same
location.

5
RESPIRATORY-System Wise 1700-by Sush and Team. 2016
Susmita, Asad, Manu, Saima, Zohaib, Savia, Shanu, Mona, Manisha, Sitara, Samreena, Sami and Komal


74. A 35yo man presents with progressive breathlessness. He gave a hx of
polyarthralgia with painful lesions on the shin. CXR: bilateral hilar lymphadenopathy.

What’s the most likely dx?
a. Bronchial asthma
b. Cystic fibrosis
c. Sarcoidosis
d. Bronchiectasis
e. Pneumonia
Q. 1. What is the key?
Q.2. What is the specific name of this condition? What is the triad?
Ans. 1. The key is C. Sarcoidosis.
Ans. 2. Lofgren syndrome. The triad is 1) Erythema Nodosum 2) Bilateral hilar
lymphadenopathy 3) Arthralgia


Sarcoidosis: (PG 186 OHCM)

6
RESPIRATORY-System Wise 1700-by Sush and Team. 2016
Susmita, Asad, Manu, Saima, Zohaib, Savia, Shanu, Mona, Manisha, Sitara, Samreena, Sami and Komal

Presentation:
Lungs are in involved in more than 90% cases of sarcoidosis and have abnormal
CXR’s with BHL (*Bilateral Hilar Lymphadenopathy) There is interstitial lung disease.
The painful skin lesion is erythema nodosum. Also look for Lupus pernio (chronic
raised hardened, often purple lesion) may be seen on the face.

Lofgren syndrome is often a part of sarcoidosis. The triad is 1) Erythema Nodosum
2) Bilateral hilar lymphadenopathy 3) Arthralgia

Sarcoidosis is a multisystem disease and can involve any system/organ

Tests: ESR is often raised. Serum ACE enzyme levels are raised in 60% of times
Plain CXR may show bilateral hilar or para-tracheal lymphadenopathy. High
resolution CT should be done. There will be restrictive pattern of disease on
pulmonary function tests.
Trans bronchial biopsy can demonstrate the presence of non-caseating
granulomata, giving a more accurate diagnosis. Bronchi alveolar lavage may also be
done.



*Bilateral hilar lymphadenopathy is a bilateral enlargement of the lymph nodes of
pulmonary hila. It is a radiographic term that describes the enlargement of
mediastinal lymph nodes and is most commonly identified by a chest x-ray.

Management

Patient with BHL sarcoidosis – not treatment needed as most recover spontaneously.

Acute Sarcoidosis ! Bed rest and NSAIDS

Corticosteroid ! Prednisolone 40mg/24hr for 4/6 weeks and then gradually
decrease over the year.

*Indication for corticosteroids: 1) Parenchymal lung disease, 2) Cardiac involvement,
3) Uveitis

93. A 48yo woman is admitted to ED with a productive cough and moderate fever.

She has often central chest pain and regurgitation of undigested food most times but

doesn’t suffer from acid reflux. These symptoms have been present for the last 3.5

7
RESPIRATORY-System Wise 1700-by Sush and Team. 2016
Susmita, Asad, Manu, Saima, Zohaib, Savia, Shanu, Mona, Manisha, Sitara, Samreena, Sami and Komal

months which affects both food and drink. A CXR shows an air-fluid level behind a

normal sized heart. What is the most likely dx?

a. Pharyngeal pouch

b. Hiatus hernia (acid reflux will be present)

c. Bulbar palsy

d. Achalasia

e. TB

Q. 1. What is the key?

Q. 2. What are the points in favour?

Ans. 1. The key is D. Achalasia.

Ans. 2. Points in favour: Aspiration pneumonia (160 ohcm) (Aspiration pneumonia

is a lung infection that develops after you aspirate (inhale) food, liquid, or vomit into

your lungs. If you are not able to cough up the aspirated material, bacteria can grow

in your lungs and cause an infection) due to retained food and fluid in esophagus.

Regurgitation of undigested food without acid reflux. Dysphagia for both food and

drink. Air-fluid level behind heart.

(PG 240) ACHALASIA - a condition in which the muscles of the lower part of the

oesophagus fail to relax, preventing food from passing into the stomach.

Why it is not hiatus hernia? Ans. Differentiating point: -i) In hiatus hernia usually you

will get associated GORD [particularly in sliding hernia which is the most common

(99%). However, in rolling hernia there may be no reflux]. ii) In hiatus hernia x-ray

8
RESPIRATORY-System Wise 1700-by Sush and Team. 2016
Susmita, Asad, Manu, Saima, Zohaib, Savia, Shanu, Mona, Manisha, Sitara, Samreena, Sami and Komal

chest may demonstrate a retro cardiac gas-filled structure rather than air-fluid level

iii) Also in hiatus hernia there may be nausea or vomiting.

Why it is not pharyngeal pouch? Ans. In pharyngeal pouch there will be halitosis (bad

breath)


SKYPE-RESPIRATORY-1700

Sush- 112, 139, 171, 174, 179



112. A 28yo man has developed a red, raised rash on trunk after playing football.
His PMH shows he had childhood asthma. The rash is becoming increasingly itchy.
What is the most appropriate tx?

a. Oral chlorpheneraime
b. Oral amoxicillin
c. IM adrenaline
d. Nebulized salbutamol (in COPD/asthma)
e. Histamine

Q. What is the key?
Q. 2. What is the diagnosis?

Ans. 1. The key is A. Oral chlorpheneramine.
Ans. 2. Diagnosis is Atopy (allergy).

Since it is an allergic reaction only 2 options are suitable. A & C. IM adrenaline is used
in anaphylactic shock which can occur due to allergy. But this is just a mild allergic
reaction here so anti histamine (chlorpheneramine) is adequate.

(Extra) Anaphylaxis presents with:
• Sudden onset and rapid progression of symptoms.
• Life-threatening airway and/or breathing and/or circulation problems
• Patient will be mostly in shock.
Mostly commonly caused by certain foods like peanuts, pulses, fish, eggs. Also by
venom (bee,wasps) and drugs like antibiotics.
Treatment:ABCDE, Oxygen, IM Adrenaline. <6yrs0.15ml, 6-12yrs 0.3ml, >12 yrs
0.5ml 1:1000

139. An old alcoholic presents with cough, fever, bilateralcavitating consolidation.


What is the most probable cause?

9
RESPIRATORY-System Wise 1700-by Sush and Team. 2016
Susmita, Asad, Manu, Saima, Zohaib, Savia, Shanu, Mona, Manisha, Sitara, Samreena, Sami and Komal

a. Gram +ve diplococcic (psumococcus)
b. Coagulase +vecocci
c. Gram –vecocci
d. AFB
e. Coagulase –vecocci

Q. 1. What is the key?
Q. 2. What is the organism?

Ans. 1. The key is B. Coagulase +vecocci.
Ans. 2. Name of organism is Staphylococcus aureus.





Pulmonary cavities are gas-filled areas of the lung in the center of a nodule, mass or
area of consolidation: causes- malignancy, infection (TB, Pneumonia)etc

Consolidation occurs when the normally air filledlung parenchyma becomes
engorged with fluid or tissue, most commonly in the setting of pneumonia.

**Sush: cavitation can occur in Staph pneumonia, klebsiellabut alcoholics have
Klebsiella: so I think Klebsiella [CHECK??]

Investigations in Respiratory medicine (more): Samson note: page 12/13 of 39

• Bilateral Cavitation- Staph pneumonia
• Upper lobe cavitation: usually in TB

• Patchy consolidation: Mycoplasma or Legionella
• Upper lobe Consolidation: TB, rarely in Klebsiella(but somewhere also
mentioned that upper lobe cavitation occurs in Klebsiella)

• Bilat interstitial shadowing: pneumocystis jiroveci



• Legionella: hotel stay, foreign travel, flu like symptoms, hyponatremia, pleural
effusion. TEST: urinary antigen. CXR shows bi-basal consolidation
• Mycoplasma Pneumonae: Rash (erythema multiforme), unusual symptoms
(abd pain, dry cough), long duration of symptoms, hyponatremia, Diagnosis by
serology. CXR: reticular-nodular shadowing or patchy consolidation
• Staphylococcal pneumonia may complicate influenza infection and is seen
most frequently in the elderly and in intravenous drug users or patients with
underlying disease. Shows bilateral cavitations.

10
RESPIRATORY-System Wise 1700-by Sush and Team. 2016
Susmita, Asad, Manu, Saima, Zohaib, Savia, Shanu, Mona, Manisha, Sitara, Samreena, Sami and Komal

• Pneumonia associated with COPD: H.influenze (more likely) or P.aeruginosa
• P.aeruginosa: Common in bronchiectasis or CF. Also causes hospital acquired
infection.
• (??) Klebsiellapneumoniae is classically in alcoholics(and cavitation occurs in
upper lobes)
• Strept pneumonia: Associated with herpes labialis. commoner in the elderly,
alcoholics, post-splenectomy, immunosuppressed and patients with chronic heart
failure or pre-existing lung disease
• Pneumocystis pneumonia (PCP) causes pneumonia in the immunosuppressed
(eg HIV). CXR may be normal or show bilateral perihilar interstitial shadowing.
Diagnosis: visualization of the organism in induced sputum, bronchoalveolar lavage,
or in a lung biopsy specimen

SO either klebsiella (gram - rod) or streptococcus as these are the ones common in
alcoholics but bilateral cavitations do point in favor of staphylococcus.

Sush: To find a chart for the bacteria in various pneumonia– there is one somewhere

171. A 50yo chronic smoker came to OPD with complaint of chronic productive
cough, SOB andwheeze. Labs: CBC=increase in PCV. CXR >6ribs seen above the
diaphragm in midclavicular line. ABG=pO2 decreased. What is the most likely dx?


a. Interstitial lung disease
b. Wegener’s granulomatosis
c. Ca bronchi
d. COPD
e. Amyloidosis

Q. 1. What is the key?
Q. 2. What are the points in favour?

Ans. 1. The key is D. COPD.
Ans. 2. Points in favour: i) Age 50 yrs ii) Chronic smoker iii) Chronic productive cough,
SOB and Wheeze iv) Raised PCV secondary to chronic hypoxaemia v) Low set
diaphragm and widened horizontal ribs vi) Hypoxaemia on ABG.

COPD:

COPD is a common progressive disorder characterized by airway obstruction (FEV1
<80% predicted; FEV1/FVC <0.7.

It includes emphysema and chronic bronchitis.

11
RESPIRATORY-System Wise 1700-by Sush and Team. 2016
Susmita, Asad, Manu, Saima, Zohaib, Savia, Shanu, Mona, Manisha, Sitara, Samreena, Sami and Komal

COPD is favoured by: •Age of onset >35yrs •Smoking (passive or active) or pollution
related •Chronic dyspnoea •Sputum production •Minimal diurnal or day-to-day
FEV1 variation.

Chronic bronchitis is defined clinically as cough,sputum production on most days for
3 months of 2 successive yrs. (Chronic bronchitis is a chronic inflammatory condition
in the lungs that causes the respiratory passages to be swollen and irritated,
increases the mucus production and damages the lungs.)
Symptoms improve if they stop smoking.

Emphysema is defined histologically as enlarged air spaces distal to terminal
bronchioles, with destruction of alveolar walls.(It is a lung condition featuring an
abnormal accumulation of air due to enlargement and destruction of the lung's
many tiny air sacs resulting in the formation of scar tissue.)


Pink puffers have inc alveolar ventilation, a near normal PaO2 and a normal or low
PaCO2. They are breathless but are not cyanosed. They may progress to type 1
respiratory failure.
Blue bloatershave decreased alveolar ventilation, with a low PaO2 and a high
PaCO2. They are cyanosed but not breathless and may go on to develop
corpulmonale. Their respiratory centresare relatively insensitive to CO2 and they rely
on hypoxic drive to maintain respiratoryeffort

Symptoms Cough; sputum; dyspnoea; wheeze.
SignsTachypnoea; use of accessory muscles of respiration; hyperinflation;
cricosternal distance (<3cm); decreased chest expansion; resonant or hyperresonant
percussion note; quiet breath sounds (eg over bullae); wheeze; cyanosis;
corpulmonale.
Complications Acute exacerbations ± infection; polycythaemia; respiratory failure;
corpulmonale (oedema; raised JVP); pneumothorax (ruptured bullae); lung
carcinoma.
Tests
• FBC:Raised PCV.
HB and PVC may often be raised – as a result of persistent hypoxaemia –
which causes a secondary polycythaemia

• CXR:Hyperinflation (>6 anterior ribs seen above diaphragm in midclavicular
line); flat hemidiaphragms; large central pulmonary arteries;
peripheralvascular markings; bullae.
• ECG: Right atrial and ventricular hypertrophy (corpulmonale).
• ABG:Reduced PaO2 ± hypercapnia.
• Lung function:obstructive + air trapping (FEV1 <80% of predicted. FEV1 : FVC
ratio <70%

TREATMENT OF CHRONIC STABLE PATIENTS.

12
RESPIRATORY-System Wise 1700-by Sush and Team. 2016
Susmita, Asad, Manu, Saima, Zohaib, Savia, Shanu, Mona, Manisha, Sitara, Samreena, Sami and Komal

Sush: other diseases mentioned here in key should be read



174. A 72yo man is receiving chemotherapy for SCLC. He has his 4th tx 8 days ago.
He has a cough with some green sputum but feels well. Temp=37.6C. Chest exam =
few coarse crepitations in the right base. HR=92bpm. CBC: Hgb=12.5g/dL,
WBC=1.1, Neutrophils=0.6, Plt=89. Sputum, urine and blood culture sent to
microbiology. What is the most appropriate management?


a. Broad spectrum antibiotics IV
b. Broad spectrum antibiotics PO
c. GCSF (in myeoproliferative dirorders, when everything low)
d. Postpone tx until bacteriology results available
e. Reassure and send home

13
RESPIRATORY-System Wise 1700-by Sush and Team. 2016
Susmita, Asad, Manu, Saima, Zohaib, Savia, Shanu, Mona, Manisha, Sitara, Samreena, Sami and Komal

Q. 1. What is the key?
Q. 2. What is the Diagnosis?
Q. 3. What is the treatment of low WBC count?

Ans. 1. The key is A. Broad spectrum antibiotics IV
Ans. 2. The diagnosis is lower respiratory tract infection.
Ans. 3. GCSF subcutaneously. [it is the treatment of chemotherapy induced
leucopenia]

SCLC: small cell lung cancer
GCSF:Granulocyte-colony stimulating factor (G-CSF or GCSF), also known as colony-
stimulating factor 3 (CSF 3), is a glycoprotein that stimulates the bone marrow to
produce granulocytes and stem cells and release them into the bloodstream.[it is the
treatment of chemotherapy induced leucopenia]Granulocytes colony stimulating
factors are used to produce neutrophils and is used in preventing sepsis but the
patient here is almost in sepsis! GCSF are mostly used in myeloproliferative
disorders.



Sush: to learn normal blood levels

Green sputum: sign of infection
Coarse creps: ?TB

Sush: Here we are suspecting impending septicemia due to reduced neutrophil and
slightly elevated temperature. Patient is immunocompromised, lung infection? In
this question the neutrophil count is 0.6 and temp is 37.6. So we should still start Iv
antibiotics as they are dangerously close the ranges given in the text above.


Use of antibiotics in neutropenia:
Treat any known infection promptly.
• If T° >38°C or T° >37.5°C on 2 occasions, >1h apart, or the patient is toxic, assume
septicaemia and start blind combination therapy—egpiperacillin–tazobactam—(+
vancomycin,if Gram +ve organisms suspected or isolated, eg Hickman line sepsis).
Check local preferences. Continue until afebrile for 72h or 5d course, and until
neutrophils >0.5≈109/L. If fever persists despite antibiotics, think of CMV, fungi (eg
Candida; Aspergillus) and central line infection.
• Consider treatment for Pneumocystis eg co-trimoxazole, ie trimethoprim
20mg/kg + sulfamethoxazole 100mg/kg/day PO/IV in 2 daily doses). Remember TB.

Avoid IM injections as they can lead to hematomas.


179. A 35yo woman had an uneventful lap chole 18h ago. She has a pulse=108bpm,
temp 37.8C. There are signs of reduced air entry at the right base but the CXR

14
RESPIRATORY-System Wise 1700-by Sush and Team. 2016
Susmita, Asad, Manu, Saima, Zohaib, Savia, Shanu, Mona, Manisha, Sitara, Samreena, Sami and Komal

doesn’t show an obvious abnormality. What is the most appropriate management
strategy?

a. Cefuroxime PO
b. Ceftriaxone IV
c. Chlorpheniramine PO
d. Chest physiotherapy
e. Reassure

Q. 1. What is the key?
Q. 2. What is the diagnosis?

Ans. 1. The key is D. Chest physiotherapy.
Ans. 2. Atelactasis


Sush:
Post operativeatelactesis occurs within 48 hours. Very common with some
pulmonary collapse occurring after abdominal or transthoracic procedure. Mucus is
retained in the bronchial tree, blocking finer bronchi, alveolar air is reabsorbed and
collapse of supplied lung segments occure. There may be secondary infection.

c/F
1. Dyspnoeic
2. Rapid pulse
3. Pyrexia (cyanosis)
4. Painful coughing (characteristic painful fruity cough)
5. Frothy/clear sputum> later purulent
6. Chest movements diminished (esp on affected side)> basal dullness, air entry
reduced with creps> CXR: opacity of involved seg, mediastenal shift to
affected side
Mx:
• Chest Physio> chest percussion and breathing exercises, passive postural
drainage
• Analgesia (intercostal block in upper abd incisions)
• Severe: bronchial catheter
• >48 hrs pyrexia>secondary chest inf>pneumococcus >augmentin (if
aspiration possible> add metro)

Prevention: stop smoking treat bronchitis, deep breathing and coughing, incentive
spirometry, not excess analgesia

Best visible on CT scan and not on chest xray. Mostly occurs as a complication of
anaesthesia.

Arrange physiotherapy and antibiotics.

15
RESPIRATORY-System Wise 1700-by Sush and Team. 2016
Susmita, Asad, Manu, Saima, Zohaib, Savia, Shanu, Mona, Manisha, Sitara, Samreena, Sami and Komal


307. A 46yo man is being treated for a pleural effusion. A chest drain has been sited
just below the
4th rib in the mid-axillary line on his right side. What single structure is at particular
risk of injury?
a. Arzygos vein
b. Diaphragm
c. Intercostal artery
d. Internal thoracic artery
e. Liver
Ans. The key is C. Intercostal artery. [Most vulnerable structure is intercostal nerve,
then intercostal artery then intercostals vein].

Chest drain is placed between 4th to 6th intercostal space anterior to the mid axillary
line. This is called safe triangle. Sometimes below 7th intercostal space when there is
loculated pleural effusion, or occasionally 2nd intercostal space in the mid clavicular
line for apical pneumothorax.
Injury to neurovascular bundle is a complication. You have to insert the needle
above the 6th rib inorder to prevent injury to 5th intercostal space neurovascular
bundle.

Pleural effusion is fliud in pelural space. Divided by theior protein conc. Into
TRANSUDDATES (<25g/L)
EXUDATES(>25g/L)

Blood….haemothorax
Pus….empyema
Chyle(lymph with fat)….chylothorax.


CAUSES..
TRANSUDATES maybe due to increased venous prerssure (cardiac failure, fluid
overload, constrictive pericarditis) OR hypoproteinemia (cirrhosis, nephrotic
syndrome, malapsorption)

EXUDATES…. Due to increased leakiness of pleural capillaires secondary to infection,
infalammation or malignancy… pneumonia, TB, RA, SLE, malignant mets.


SYMPTOMS… asymotiomatic – or dyspnea, pleuritic chest pain.

SIGNS…
Decreased expansion
Stony dull percussion
Diminished breath sounds
Tactile vocal fremitus and vocal resonance decreased.

16
RESPIRATORY-System Wise 1700-by Sush and Team. 2016
Susmita, Asad, Manu, Saima, Zohaib, Savia, Shanu, Mona, Manisha, Sitara, Samreena, Sami and Komal

Tracheal deviation away from effusion.
Signs of associated disease or aspiration marks



TESTS
CXR..blunt costophrenic angles
USG

DIAGNOSTGIC ASPIRATION… send fluid for clinical cghemisatry, bacteriology,
cytology, immunology.


PLEURAL BIOPSY…If fluid analysis inconclusive


MANAGEMENT….is of the underlying cause
DRAINAGE
Pleurodesis
Surgery…persistent collection and increasing pleural thickness on USG require
surgery






335. A 48yo woman who has been taking medications for asthma for a long time has
now presented
with decreasing vision. What is the most probable cause for her decrease in vision?
a. Inhaled salbutamol
b. Inhaled steroids
c. Aminophylline
d. Beta-blockers
e. Oral steroids

Q. 1. What is the key?
Q. 2. Justify the key.

Ans. 1. The key is E. Oral steroid.

Ans. 2. Prolonged steroid use leads to cataract formation, glaucoma and eye
diseases.

17
RESPIRATORY-System Wise 1700-by Sush and Team. 2016
Susmita, Asad, Manu, Saima, Zohaib, Savia, Shanu, Mona, Manisha, Sitara, Samreena, Sami and Komal


Inhaled salbutamol has side effects of tremors, anxiety, decreased K.

Inhaled steroids can cause oral candidiasis.

Aminophylline can cause GI upsets, arrhythmias or fits.

Beta blockers can cause bronchoconstriction etc.

So prolonged use of oral steroids is the correct answser.

Ohcm page 371,,,side effects of steroid use.

ENT.345. Pt presented with hemoptysis 7d post-tonsillectomy. What is the next


step?
a. Packing
b. Oral antibiotics + discharge
c. Admit + IV antibiotics
d. Return to theatre and explore
e. Ice cream and cold fluids

Ans. The key is C. Admit + IV antibiotic. [infection is a common cause of secondary
haemorrhage, haemoorrhage occurring ].

377. A 70yo man admits to asbestos exposure 20yrs ago and has attempted to quit
smoking. He has noted weight loss and hoarseness of voice. Choose the single most
likely type of cancer a.w risk
factors present.
a. Basal cell carcinoma
b. Bronchial carcinoma
c. Esophageal carcinoma
d. Nasopharyngeal carcinoma
e. Oral carcinoma

B. Bronchial carcinoma.
[Asbestos exposure is a risk factor for lung cancer and also has a synergistic effect
with cigarette smoke].

18
RESPIRATORY-System Wise 1700-by Sush and Team. 2016
Susmita, Asad, Manu, Saima, Zohaib, Savia, Shanu, Mona, Manisha, Sitara, Samreena, Sami and Komal

#. Conditions related to asbestos exposure: i) Pleural plaques (after a latent period of
20-40 yrs) ii) Pleural thickening iii) Asbestosis (latent period is typically 15-30 yrs) iv)
Mesothelioma (prognosis is very poor) v) Lung cancer.

387. A 54yo pt 7 days after a total hip replacement presents with acute onset
breathlessness and
raised JVP. Which of the following inv will be most helpful in leading to a dx?
a. CXR
b. CTPA
c. V/Q scan
d. D-Dimer
e. Doppler US of legs

The key is B. CTPA.



The patient has a +ve two level PE Wells score (if it was negative we should do D-
Dimer) and there is no renal impairment or history suggestive of allergy to contrast
media (if these present we should have go for VQ scan) the investigation of choice is
PTCA. NICE guideline.

According to ohcm, the question of V/Q SCAN or CTPA being a better choice of
investigation in PE, has no clear answer. CTPA is sensitive and specific (>90%). New
multidetector CT systems give faster scanning time and good subsegmental imaging,
requiring less IV contrast, beneficial for patients with renal and cardiac impairment.

1st line is CTPA (which can show clots down to the 5th order pulmonary arteries) and
venous phase CT of the leg veins and pelvis.
CTPA’s advantages are that it is more readily available and that it can show other
lesions.

Bilateral leg ultsounds or venograms may also be sufficient to CONFIRM, but not
exclude, a PE in pateints with a co existing clinical DVT.





924. A 33yo man has a temp=38.5C, cough and chest pain on the right si e on
inspiration. He also has purulent sputum. What is the most likely organism to cause
pneumonia in this pt?
a. Gram +ve diplococci

19
RESPIRATORY-System Wise 1700-by Sush and Team. 2016
Susmita, Asad, Manu, Saima, Zohaib, Savia, Shanu, Mona, Manisha, Sitara, Samreena, Sami and Komal

b. Coagulase +ve cocci
c. PCP cold agglutinins
d. AFB
e. Gram –ve diplococci

Key : c ....... corrected : a
Clincher : no specific pointers here

The most common cause of CAP strept.pneumonia which is a gram +ve diplococci




929. A 65yo man presents with significant weight loss and complains of cough, SOB
and chest pain. Exam: left pupil constricted, drooping of left eyelid. What is the most
likely dx?
a. Pancoast tumor
b. Thoracic outlet syndrome
c. Cervical rib
d. Pneumonia
e. Bronchogenic ca

a. Pancoast tumor
Classically caused by an apical (superior pulmonary sulcus) malignant neoplasm of
the lung. The neoplasm is usually bronchogenic in origin (most commonly squamous
cell carcinoma, sometimes adenocarcinoma and large-cell carcinoma).
• An ipsilateral invasion of the cervical sympathetic plexus leading to Horner's
syndrome (miosis, enophthalmos, ptosis; in 14-50% of patients).
• Ipsilateral reflex sympathetic dystrophy may occur.
• Shoulder and arm pain (brachial plexus invasion C8-T2) leading to wasting of
the intrinsic hand muscles and paraesthesiae in the medial side of the arm.
• Less commonly, unilateral recurrent laryngeal nerve palsy producing
unilateral vocal cord paralysis (hoarse voice ± bovine cough), and/or phrenic
nerve involvement.
• There may be arm oedema secondary to the compression of blood vessels.






935. A 62yo man has been smoking about 15 cigarettes/day for 45yrs, and has been
working as a
builder since he was 24yo. He presents with chest pain, SOB, weight loss. CXR shows
bilateral
fibrosis and left side pleural effusion. What is the best inv that will lead to dx?
a. CXR

20
RESPIRATORY-System Wise 1700-by Sush and Team. 2016
Susmita, Asad, Manu, Saima, Zohaib, Savia, Shanu, Mona, Manisha, Sitara, Samreena, Sami and Komal

b. Pleural fluid aspiration of cytology
c. MRI
d. Pleural biopsy
e. CT

d. Pleural biopsy
Only biopsy confirms carcinoma
Ct is the next step not the best step towards the diagnosis. The best work up leading
to diagnosis should b pleural biopsy,
Asbestosis predisposing to mesothelioma and therefore pleural biopsy
Pleural fluid: straw coloured or blood stained. Cytological analysis occasionally leads
to the diagnosis but a pleural biopsy is usually required.








936. During a basketball match, one of the players suddenly collapsed to the ground
with coughing and SOB. What is the inv of choice?
a. CXR
b. CT
c. MRI
d. V/Q scan
e. CTPA

a. CXR
case of spontaneous pneumothorax...xray chest ...first
Basketball player... tall height.... more chances of apical subpleural blebs... its
SPONTANEOUS PNEUMOTHORAX.. so CXR is best



963. A 73yo man who was a smoker has quit smoking for the past 3yrs. He now
presents with hoarseness of voice and cough since past 3wks. XR: mass is visible in
the mediastinum. What is the best inv to confirm the dx?
a. Bronchoscopy
b. Thoracoscopy
c. US
d. CT thorax
e. LN biopsy

Ans: E

21
RESPIRATORY-System Wise 1700-by Sush and Team. 2016
Susmita, Asad, Manu, Saima, Zohaib, Savia, Shanu, Mona, Manisha, Sitara, Samreena, Sami and Komal

Reason : the patient seems to be affecting from Bronchogenic Carcinoma, The Key
diagnosis as rccomended by NICE
Xray
CT
Bronchoscopy with Biopsy
Biopsy of a convenient metastatic site should be performed if this is easier than
biopsy of a primary site . (Biopsy remains the best investigation to confirm the
diagnosis)
NICE strongly recommends a new imaging test,18F-deoxyglucose positron emission
tomography (FDG-PET), to help stage tumours

Treatment
Small cell tumours are usually treated with 4-6 cycles of multi-drug platinum-based
chemotherapy with the possibility of added radiotherapy in limited stage disease.
Most Rapidly Growing+Worst Prognosis

Staging of the disease and the patient’s overall fitness and exercise ability will
determine the type of treatment chosen
Surgery , Radiotherapy, Radical Radiotherapy , RADIO+ Chemo

Respiratory questions 396 400 441 455 522 1462 1491


Q 396. A 56 yo lady with lung cancer presents with urinary retention, postural
hypotension, diminished reflexes and sluggish papillary reaction. What is the
most likely explanation for her symptoms?
a. Paraneoplastic syndrome
b. Progression of lung cancer
c. Brain metastasis
d. Hyponatremia
e. Spinal cord compression
Clues in question
Age of patient
Type of cancer
Complex symptoms

Q 400. A child suffering from asthma presents with Temp 39C,
drooling saliva on to the mother’s lap, and taking oxygen by mask.
What sign will indicate that he is deteriorating?
f. Intercostal recession
g. Diffuse wheeze
h. Drowsiness
Criteria for assessing severity of attack
Severe attack
. inability to complete sentence
.pulse>110 bpm

22
RESPIRATORY-System Wise 1700-by Sush and Team. 2016
Susmita, Asad, Manu, Saima, Zohaib, Savia, Shanu, Mona, Manisha, Sitara, Samreena, Sami and Komal

.RR >25/min
.PEF 33-50% of predicted
Life threatening attack
silent chest . cyanosis . bradycardia .exhaustion . PEF <33% . feeble
respiratory effort . confusion
Signs which occur during an asthma attack include the use of
accessory muscles of respiration
The wheezing is most often when breathing out
Q 441. A 16yo teenager was brought to the ED after being stabbed on the
upper right side of his back. Erect CXR revealed homogenous opacity on the
lower right lung, trachea was centrally placed. What is the most probable
explanation for the XR findings?
i. Pneumothorax
j. Hemothorax
k. Pneumonia
l. Tension pneumothorax
m. Empyema




2. Q 455. A 48yo farmer presented with fever, malaise, cough and SOB. Exam:
tachypnea, coarse end-inspiratory crackles and wheeze throughout, cyanosis.
Also complaint severe weight loss. His CXR shows fluffy nodular shadowing
and there is PMN leukocytosis. What is the single most appropriate dx?
a. Ankylosing spondylitis
b. Churg-strauss syndrome
c. Cryptogenic organizing
d. Extrinsic allergic alveolitis
e. Progressive massive fibrosis
Clues in question
Patient’s occupation
Weight loss
leukocytosis



Q 522. An 8yo boy has longstanding asthma. He has admitted with a severe
episode and is tired and drowsy. He has not improved on oxygen, inhaled B2
agonist and IV hydrocortisone. CXR shows bilateral hyperinflation. He is too
breathless to use a peakflow meter and is O2 sat <90%. What is the single
most appropriate inv?
f. CBG
g. CXR
h. CT chest
i. Pulse oximetry
j. Spirometry

23
RESPIRATORY-System Wise 1700-by Sush and Team. 2016
Susmita, Asad, Manu, Saima, Zohaib, Savia, Shanu, Mona, Manisha, Sitara, Samreena, Sami and Komal



Q 1462. A 50yo man presents with itching after hot shower with dizziness,
chest pain after exercise. Exam: splenomegaly. What is the single most likely
causative factor?
k. ALL
l. Lymphoma
m. Polycythemia
n. Scabies
o. Eczema
Clues in question
. itching after hot bath
Dizziness . tinnitus . visual disturbances .headache due to
hyperviscosity


Q 1491 .A 48yo woman is admitted to the ED with a productive cough and
mod fever. She often has central chest pain and she regurgitates undigested
food most of the time but doesn’t suffer from acid reflus. These symptoms
have been present for the last 3.5m which affects her daily food intake. CXR:
air-fluid level behind a normal sized heart. What is the single most likely dx?
p. Pharyngeal pouch
q. Hiatus hernia
r. Bulbar palsy
s. Achalasia
t. TB




547,548,550,581,593 extra information

547. A 50yo woman returned by air to the UK from Australia. 3days later she
presented with sharp chest pain and breathlessness. Her CXR and ECG are normal.
What is the single most
appropriate inv?
a. Bronchoscopy
b. Cardiac enzymes
c. CT
d. MRI
e. Pulse oximetry
f. V/Q scan
g. CTPA

g. CTPA
Long flight and sharp chest pain along with breathlessness points towards PE

24
RESPIRATORY-System Wise 1700-by Sush and Team. 2016
Susmita, Asad, Manu, Saima, Zohaib, Savia, Shanu, Mona, Manisha, Sitara, Samreena, Sami and Komal

As per NICE guidelines the most appropriate investigation is CTPA if ur suspecting PE.
V/Q scan is preferred in only few situation like pregnancy, Ckd patients, or ctpa n/a.
U have to do wells scoring of the patient and if its > 4, u straight away do CTpa ..
Dont even wait for d dimer. Definitely CTPA in this case

medical student textbook triad of pleuritic chest pain, dyspnoea and haemoptysis.
computed tomographic pulmonary angiography (CTPA) is now the recommended
gold standard

Management:
Low molecular weight heparin (LMWH) or fondaparinux should be given initially
after a PE is diagnosed.An exception to this is for patients with a massive PE where
thrombolysis is being considered. In such a situation unfractionated heparin should
be used.
• a vitamin K antagonist (i.e. warfarin) should be given within 24 hours of the
diagnosis
• the LMWH or fondaparinux should be continued for at least 5 days or until the
international normalised ratio (INR) is 2.0 or above for at least 24 hours, whichever is
longer, i.e. LMWH or fondaparinux is given at the same time as warfarin until the INR
is in the therapeutic range
• warfarin should be continued for at least 3 months.
• NICE advise extending warfarin beyond 3 months for patients with unprovokedPE.
• for patients with active cancer NICE recommend using LMWH for 6 months

Thrombolysis
• thrombolysis is now recommended as the first-line treatment for massive PE
where there is circulatory failure (e.g. hypotension). Other invasive approaches
should be considered where appropriate facilities exist


Saima

Chest x Ray finding for PE is low vascular markings and wedge shape infarct. OHCM
page 828. For massive embolism use alteplase 10 mg IV over 1 min then 90 mg over
2 hours.
548. A tall thin young man has sudden pain in the chest and becomes breathless
while crying. What is the single most appropriate inv?
a. Cardiac enzymes
b. CXR
c. CT
d. ECG
e. V/Q scan

b. CXR
spontaneous pneumothorax .most often in young thin male ..due to rupture of
bullous emphysema
patient may have marfans syndrome, or alpha-1 antitrypsin deficiency

25
RESPIRATORY-System Wise 1700-by Sush and Team. 2016
Susmita, Asad, Manu, Saima, Zohaib, Savia, Shanu, Mona, Manisha, Sitara, Samreena, Sami and Komal


Secondary pneumothorax

Recommendations include:
• if the patient is > 50 years old and the rim of air is > 2cm and/or the patient is short
of breath then a chest drain should be inserted.
• otherwise aspiration should be attempted if the rim of air is between 1-2cm. If
aspiration fails (i.e. pneumothorax is still greater then 1cm) a chest drain should be
inserted. All patients should be admitted for at least 24 hours
• if the pneumothorax is less the 1cm then the BTS guidelines suggest giving oxygen
and admitting for 24 hours

Saima

For tension pneumothorax don't wait for chest x Ray. Start alleviating the symptoms
straight away. First aspirate the air to make it into normal pneumothorax by 16 G
cannula in the 2nd intercostal space. Then do the chest drain.


550: 550. A 55yo woman with a persistent cough and hx of smoking develops left
sided chest pain
exacerbated by deep breathing with fever and localized crackles. What is the single
most
appropriate dx?
a. Dissecting aneurysm
b. Pericarditis
c. Pneumonia
d. Pneumothorax
e. Pulmonary embolism

c. Pneumonia

dissec aneurysm will have a sharp pain radiating to the back.
pericarditis has similiar features , but i dont see a cardiac cause.
pneumothorax is seen in smokers , as a complication to COPD , but it wont have
fever and above features. ( reduced air entry is seen in it, with hyper resonance on
percussin ).
pulm embloism also seems unlikely , should have travel history .
looks like pneumonia then - pain on inspiration ( pleural pain ), fever , cough, and
crackles !

Klebsiella pneumoniae is classically in alcoholics

Streptococcus pneumoniae (pneumococcus) is the most common cause of
community-acquired pneumonia

Characteristic features of pneumococcal pneumonia

26
RESPIRATORY-System Wise 1700-by Sush and Team. 2016
Susmita, Asad, Manu, Saima, Zohaib, Savia, Shanu, Mona, Manisha, Sitara, Samreena, Sami and Komal

• rapid onset
• high fever
• pleuritic chest pain
• herpes labialis

Management

CURB-65 criteria of severe pneumonia
• Confusion (abbreviated mental test score <= 8/10)
• Urea > 7 mmol/L
• Respiratory rate >= 30 / min
• BP: systolic <= 90 or diastolic <= 60 mmHg
• age >= 65 years

Patients with 3 or more (out of 5) of the above criteria are regarded as having a
severe pneumonia

The British Thoracic Society published guidelines in 2009:
• low or moderate severity CAP: oral amoxicillin. A macrolide should be added for
patients admitted to hospital
• high severity CAP: intravenous co-amoxiclav + clarithromycin OR cefuroxime +
clarithromycin OR cefotaxime + clarithromycin


Saima
Points why it's not pericarditis:
• It usually manifest as a sharp stabbing pain which radiate to scapula or arm. It
get worse in supine position, coughing or by deep inspiration and relieved by
leaning forward or sitting up.
• Pericardial rub can be heard.


581: A 26yo woman being treated for a carcinoma of the bronchus with steroids
presents with
vomiting, abdominal pain and sudden falls in the morning. What is the most specific
cause for
her symptoms?
a. Steroid side effects
b. Postural hypotension
c. Adrenal insufficiency
d. Conn’s disease
e. Cushing’s disease

c. Adrenal insufficiency

27
RESPIRATORY-System Wise 1700-by Sush and Team. 2016
Susmita, Asad, Manu, Saima, Zohaib, Savia, Shanu, Mona, Manisha, Sitara, Samreena, Sami and Komal

Streoids causing suppression of acth. In turn causing mineralcorticoid deficiency so
adrenal insufficiency
high dose sterods suppresss adrenals...cause
hyponatraemia..hypotension..hypoglycaemia..hyperkalaemia
Exogenous steroids can suppress the pituitary adrenal axis leading to adrenal
insufficiency. Symptoms include weakness, anorexia, dizzy,
Faints,nausea,vomiting,abd pain
Steroid does not cause vomit or falls so A can be excluded.
Postural hypotension does not cause abd. Pain so B excluded .
Conn's syndrome is hyperaldosteronism which would cause hypernatremia and
hypokalemia and hypertension.
So D excluded
Cushing causes hypertension so E Excluded
C is the right answer cuz adrenal insufficiency due to prolonged steroid intake would
cause addison syndrome which is hyponatremia hypotension abdominal pain .

Saima
Any exogenous steroid for long term use suppress the pituitary adrenal axis esp in
sepsis and malignancy.
Think of “ADDISON” in all with unexplained vomiting and abdominal pain.


593: A 55yo man has weight loss, dyspnea and syncope. He smokes 20
cigarettes/day. Inv confirms squamous cell carcinoma in the left bronchus. What is
the single most likely biochemical
abnormality to be a/w the condition?
a. Hypercalcemia
b. Hyperkalemia
c. Hypoernatremia
d. Hypocalcemia
e. Hypomagnesium

a. Hypercalcemia
Squamous cell ca causes hypercalcemia...whereas small cell ca causes SIADH
paraneoplastic syndrome causing ectopic production of parathyroid hormone-
related protein (PTHrP), resulting in "hypercalcemia"

There are three main subtypes of non-small cell lung cancer:

Squamous cell cancer
• typically central
• associated with parathyroid hormone-related protein (PTHrP) secretion →
hypercalcaemia
• strongly associated with finger clubbing

Adenocarcinoma
• typically peripheral

28
RESPIRATORY-System Wise 1700-by Sush and Team. 2016
Susmita, Asad, Manu, Saima, Zohaib, Savia, Shanu, Mona, Manisha, Sitara, Samreena, Sami and Komal

• most common type of lung cancer in non-smokers, although the majority of
patients who develop lung adenocarcinoma are smokers

Large cell lung carcinoma
• typically peripheral
• anaplastic, poorly differentiated tumours with a poor prognosis
• may secrete β-hCG

Saima
small cell lung cancer cause SIADH which result in increase ADH and therefore
hyponatremia.
It also cause ACTH secretion and Cushing syndrome.
Squamous excites parathyroid hormone and therefore cause hypercalcemia. OHCM
page 170.

Sami
614. A 6yo girl has had 2 short episodes of cough and wheeze over the last 12m.
These
2 acute
episodes responded quickly to bronchodilator, she has no symptoms or abnormal
physical signs.
She has slight eczema and her mother has asthma. What is the single most
appropriate
inv?
a. CXR
b. Peak flow rate diary
c. Pulse oximetry
d. Spirometry
e. Sweat test
Answer. D spirometry
Source: Davidson’s P 668 The diagnosis of asthma is predominantly clinical and
based on a characteristic history. Supportive evidence is
provided by the demonstration of variable airflow
obstruction, preferably by using spirometry (Box 19.21)
to measure FEV1 and VC. This identifies the obstructive
defect, defines its severity, and provides a baseline for
bronchodilator reversibility (Fig. 19.19). If spirometry is
not available, a peak flow meter may be used.
OHCM 165:
Spirometry measures functional lung volumes. Forced expiratory volume in 1s
(FEV1) and forced vital capacity (FVC) are measured from a full forced expiration
into a spirometer (Vitalograph®); exhalation continues until no more breath can
be exhaled. FEV1 is less effort-dependent than PEF. The FEV1/FVC ratio gives a good
estimate of the severity of airflow obstruction; normal ratio is 75–80%. See BOX.
Obstructive defect (eg asthma, COPD) FEV1 is reduced more than the FVC, and the
FEV1/FVC ratio is <75%.
Restrictive defect (eg lung fibrosis) FVC is decreased and the FEV1/FVC ratio is

29
RESPIRATORY-System Wise 1700-by Sush and Team. 2016
Susmita, Asad, Manu, Saima, Zohaib, Savia, Shanu, Mona, Manisha, Sitara, Samreena, Sami and Komal

unchanged or increased. Other
causes: sarcoidosis; pneumoconiosis, interstitial pneumonias; connective tissue
diseases; pleural effusion; obesity; kyphoscoliosis; neuromuscular problems.
Peak expiratory flow (PEF) is measured by a maximal forced expiration through
a peak flow meter. It correlates well with the forced expiratory volume in 1 second
(FEV1) and is used as an estimate of airway calibre, but is more effort-dependent.
PEF helps: in acute attack to assess the severity of acute asthmatic attack. In chronic
attack to monitor (any other use?)
Sever attack PEF 33-50%. Life threatening attack < 33% of predicted
OHCM P 172 Chronic asthma: PEF monitoring (p156): a diurnal variation of >20%
on ≥3d a wk for 2wks. Spirometry: obstructive defect (decrease FEV1/FVC, increase
RV p156); usually
≥15% improvement in FEV1 following Beta2 agonists or steroid trial.
Dr. Rabia d. Spirometry
Diagnostic test for asthma is Spirometry. And if it was a known asthmatic case then
our investigation wud be
pefr....if the ques mentioned something abt known asthma on beta agonist and or
steroid...yt having
symptoms and exacerbations provided she is taking inhaler properly...best advice
wud be to maintain a pef
diary so tht we can adjust treatment options.
Spirometry
If FEV1/ FVC > 80% ( Restrictive lung disease)
If FEV1/FVC < 80% ( obstructive lung disease)
If reversible > 12% after brochochodilator ( reversibilty test) its asthma otherwise
copd
Spirometry
With older children with an intermediate probability of asthma, diagnostic tests such
as
PEFR and forced expiratory volume in one second (FEV1) can provide objective
measures of airways obstruction but these may be normal between episodes of
bronchospasm and provide poor discrimination with other conditions that also cause
airways obstruction.Spirometry is usually possible from about 5 years old,


619. A 14yo boy with asthma suddenly developed chest pain and increasing
breathlessness during a game of football. When seen in the ED he was not cyanosed.

30
RESPIRATORY-System Wise 1700-by Sush and Team. 2016
Susmita, Asad, Manu, Saima, Zohaib, Savia, Shanu, Mona, Manisha, Sitara, Samreena, Sami and Komal

He has reduced breath sounds on the right side. His oxygen saturation is 94% on air.
What is the single most appropriate
inv?
a. Capillary blood gases
b. CXR
c. CT chest
d. Exercise challenge
e. MRI chest


Answer: b. CXR
Refer to Q 60 Dr. As’ad’s notes

Asthma is one of the Causes of pneumothorax OHCM P. 182
[Asthma is a predisposing factor for spontaneous pneumothorax. The
presentation indicates pneumothorax for which most appropriate investigation is
CXR].
Young thin male, sudden onset of chest pain and sob during sport > pneumothorax
ENT 629. A teacher had a respiratory infection for which she was prescribed
antibiotics. After
the
antibiotic course when she rejoined school, she lost her voice completely. What is
the
single
most appropriate dx?
a. Recurrent laryngeal nerve palsy
b. Angioedema
c. Laryngeal obstruction by medication
d. Laryngitis
e. Functional dysphonia/vocal cords


Answer : E. Functional dysphonia/vocal cords
Dr. Rabia
A.Recurrent laryngeal nerve because major reasons are it's either trauma via
surgery for thyroid or any neck surgery, tumour
B. Angioedema because it presents acutely with in minutes and usually superficial ie
visible sites though also involve deeper respiratory structures. Drug's notorious to
cause
angioedema are A.C.E.i and in rare cases A.R.B.s
C. Laryngeal obstruction because it will eventually block your breathing
D.Laryngitis is an infection for which doctors do prescribe antibiotics
Antibiotics rarely cause voice loss.
Where no organic cause is found - a diagnosis of exclusion.
OHCS P 568
Functional disorders of speech articulation: (ie
cause unknown). Aphonia: Phonation yields no

31
RESPIRATORY-System Wise 1700-by Sush and Team. 2016
Susmita, Asad, Manu, Saima, Zohaib, Savia, Shanu, Mona, Manisha, Sitara, Samreena, Sami and Komal

response (or a whisper) in seemingly normal cord
adductors, eg in young women at times of stress (NB: there are many functional
voice disorders which may result in laryngeal oedema ± nodules.) A good
differentiating test is to ask patients to cough (needs functional adductors).
It is a diagnosis by exclusion, eg allergic reactions may cause sudden aphonia, so
don’t assume a functional disorder without laryngoscopy. Treatment:
Speech therapy is the best, with attention to emotional factors which may be
present. Spasmodic dysphonia: Strained, effortful, hoarse voice + tremors,
jerky voice onset, intermittent voice breaks, breathy spasms, hypernasality,
failure to maintain voice.186 Children with functional speech disorders have
difficulty with specific speech sounds (eg /r/, /s/, /z/, /r/, /l/ and/or ‘th’). Try
to distinguish articulation disorders from phonological delay, consistent and
varying (‘inconsistent’) phonological disorders and speech dyspraxia.
Differential: Before saying ‘no cause can be found’, consider generalized infiltrating
entities of the larynx, such as hyperkeratosis ( smoking, alcohol abuse,
pollution), leukoplakia, also granulomata, papillomata, polyps, and cysts.
691. A 16yo girl has been unwell for 5days with malaise, headache and dry cough.
She has a few
crackles in her chest. Her CXR shows patchy consolidation in the lower lobes. What is
the single
most likely causative organism?
a. Cold agglutinins
b. Gram –ve diplococci in sputum
c. Gram +ve diplococcic in sputum
d. Serology for legionella
e. Sputum staining for mycobacterium TB
A. Cold agglutinin
A and D. Atypical Pneumonia: Dry cough (mycoplasma)
OHCM P 162
Mycoplasma: occurs in epidemic every 4 y, patchy consolidation on CXR and cold
agglutinin, Complication: skin rash (erythema multiforme) Rx Clarithromycin,
Doxycyline or Fluoroquinolone
Legionella: Dry cough, history of travel and staying at a hotel, CXR patchy
consolidation, confusion, diarrhea and could cause hypoNa+, Rx clarithromycin,
fluoroquinolone
Pneumocystis Jiroveci (previously Carinii):
Immunosupression ( HIV, Drug Abuse) homosexual or African
Weight loss and lymphoadenopathy , CXR bilateral perihilar lymphoadenopathy,
Rx co-triomxazole
Chlamydophila pneumoniae is the commonest chlamydial infection. Person-
toperson spread occurs causing a biphasic illness: pharyngitis, hoarseness, otitis,
followed by pneumonia. Diagnosis: Chlamydophila complement fixation test, PCR
invasive samples.8 Treatment: doxycycline or clarithromycin.

Clamidya psittacci: Hx of birds contact Rx Clarithromycin and doxycycline

32
RESPIRATORY-System Wise 1700-by Sush and Team. 2016
Susmita, Asad, Manu, Saima, Zohaib, Savia, Shanu, Mona, Manisha, Sitara, Samreena, Sami and Komal

B. Gram –ve diplococci in sputum example Haemophilus Influenza usually in
COPD and bronchipctasis Rx Clarithromycine and Doxycyline
C. Gram +ve diplococcic in sputum example streptococcus Rx penicillin ;
NOTICE: Staphillococcus is a gram positive cluster of cocci (not diplococcic)
(CXR bilateral cavitation consolidation) Rx fluoxocilline (MRSA Rx
Vancomycine) both are more severe with sputum
E..TB alcoholic, homeless, or from Asia or Afric, 3 weeks history of productive
cough. Rx Rifampicin, Isoniazide, Pyridoxine, pyrazineamide, (Ethambutol or
sterptomycine)

Also: Pseudomonas Aeruginosa: COMMONLY causes infection in COPD and
Bronchioctasis Rx antipseudomonal Penicillin, ceftazidine, merepenem,
ciprofloxacin + aminoglycoside
ENT 700. A mother presents with her 3yo son who has indistinct nasal speech. He
snores at night and has
restless sleep. He is tired by day. What is the best management strategy?
a. Arrange hearing test
b. Assess development milestones
c. Refer to ENT surgeon
d. Refer to speech therapist
e. MRI brain

Answer C Dr. Rabia
The symptoms points to adenoid hypertrophy so for confirmation refer to ENT
surgeon
Swollen adenoids may not cause any symptoms or problems. However, symptoms
may develop in some cases, especially if the adenoids become very large. Possible
symptoms include the following:
• Breathing through the nose may be noisy or rattly. This may get worse and
cause difficulty breathing through the nose. The child then mainly mouth
breathes.
• A constantly runny nose.
• Snoring at night. In severe cases sleep may be disrupted by the blocked nose
and there is difficulty with breathing.
• Swollen adenoids may block the entrance of the Eustachian tube. This is the
tube that goes from the back of the nose to the middle ear. It normally allows
air to get into the middle ear. If this tube is blocked it may contribute to the
formation of glue ear


702. A 50yo man has had hoarseness of voice and drooping eyelid for 2m. a mass is
palpable in the right supraclavicular fossa. He smokes 20 cigarettes/day for the last
30yrs. What is the most likely dx?
a. Carcinoma larynx
b. Carcinoma thyroid
c. Carcinoma right bronchus

33
RESPIRATORY-System Wise 1700-by Sush and Team. 2016
Susmita, Asad, Manu, Saima, Zohaib, Savia, Shanu, Mona, Manisha, Sitara, Samreena, Sami and Komal

d. Mesothelioma
e. Pancoast tumor


Answer: (e) Pancoast tumor OHCM pg: 722
Pancoast syndrome: apical lung cancer
" The growing tumor can cause compression of a brachiocephalic vein,
subclavian artery, phrenic nerve, recurrent laryngeal nerve, vagus nerve or
sympathetic ganglion
" History (Hx): smoking
" Most Pancoast tumors are non-small cell cancers
Signs&Symptoms (S/S):
• compression of sympathetic plexus in the neck = Horner’s syndrome (specific
for Pancoast tumor) [miosis (pupil constriction), enophthalmos (sunken eye),
ptosis (drooping eyelid) + anhydrosis (ipsilateral loss of sweating)]
• compression of recurrent laryngeal nerve = hoarse voice/ bovine cough from
the vocal cord palsy
• Involvement of the C8-T1 of the brachial plexus = weakness of small muscles
of the hand/ arm pain
• Cancer general s/s [OHCM pg: 170]: Cough, Haemoptysis, Dyspnoea, Chest
pain, Recurrent or slowly resolving pneumonia, Anorexia, Weight loss,
Cachexia, anaemia,

Not
Carcinoma larynx: lump would be in the neck, there will be hoarseness of voice, sore
throat
Carcinoma thyroid: nodule in the thyroid region of the neck, hoarseness of voice
Mesothelioma: hx of asbestos exposure
Carcinoma of R.bronchus


704. A 19yo man has exercise induced asthma. This has prv been controlled using a
salbutamol inhaler as req, but he now gets attacks with exercise. What is the single
most appropriate tx?
a. Regular salbutamol
b. Regular salbutamol and budesonide
c. Sodium Cromoglycate
d. Oral steroid
e. Inhaled steroid


Answer: (c) Sodium cromoglycate OHCM pg: 174
Cromoglycate (mast cell stabilizer): used as prophylaxis (not acutely) in mild and
exercise-induced asthma (always inhaled), especially in children.
Not
A, B D or E: Patient (pt) is already on salbutamol and is not working.

34
RESPIRATORY-System Wise 1700-by Sush and Team. 2016
Susmita, Asad, Manu, Saima, Zohaib, Savia, Shanu, Mona, Manisha, Sitara, Samreena, Sami and Komal



743. An 8yo boy dx with asthma is on salbutamol and beclomethasone. However, he
wakes up at night due to his symptoms. What is the next appropriate management?
a. LABA
b. High dose steroid
c. Aminophylline
d. Oral prednisolone
e. Sodium cromoglycate


Answer: (a) LABA OHCM pg: 174
Pt. is getting step 2 treatment (tx): SABA (short-acting β2 agonist) – salbutamol +
inhaled steroid – beclomethasone. Still the pt. is getting nocturnal symptoms and
therefore need to step up to step 3 tx. Which is LABA (long acting β2 agonist) eg:
salmeterol

Discussion (Manu, Shanu, Asad and Sami/Osamah): the


controversial point here is in step 2 of the management, we could
increase the dose of beclamethasone up to 800 mg (ie option b.)
before switching to step 3 of Long acting beta agonist(ie option a).
So we’ve concluded that “high dose steroid” does not necessarily
mean “increasing the dose of beclomethasone”. Hence the answer
A is more probable. Also the patient has nocturnal symptoms
despite being on salbutamol and beclamethasone which is an
indication for LABA

OHCS 164: In the third step if the child is more than 5 yr add LABA. If uncontrolled
then incresae the dose of steroid

Extras: SE of salbutamo: hypo K+, and of course tachyartyhmia,


tremor and anxiety

long acting b agonist ie salmetro helps with nocturnal symptoms :


SE same as sblutamol + paradoxical bronchospasm


OHCM pg: 174; Doses on OHCM pg: 175
Management of Asthma: moving up the step if needed or down if control is good for
>3 months
Step 1: SABA inhaler eg: salbutamol, as required for symptom relief
If not controlled or used more than once daily or night symptoms, go to step 2

35
RESPIRATORY-System Wise 1700-by Sush and Team. 2016
Susmita, Asad, Manu, Saima, Zohaib, Savia, Shanu, Mona, Manisha, Sitara, Samreena, Sami and Komal

Step 2: add standard dose inhaled steroid (or according to disease severity and then
titrate accordingly) eg: beclometasone

Step 3: add LABA eg: salmeterol. If benefit- but still inadequate control: increase
dose of beclomethasone. If no effect of LABA, stop it. Review Dx: leukotriene
receptor antagonist or oral theophylline may be tried

Step 4: consider trials of: beclomethasone; modified-release oral theophylline;
modified-release oral β2 agonist; oral leukotriene receptor antagonist, in
conjunction with previous therapy. Modified-release oral β2 agonist tablets.

Step 5: add regular oral prednisolone. Refer to asthma clinic

776. A 68yo man with DM and HTN was noted to have cholesterol level of
3.4mmol/l. he was also noted to have microalbuminuria. What is the best drug to
add to his regimen?
a. ACEi
b. Statin
c. Amylodipine
d. Biguanides


Answer: (a) ACEi OHCM pg: 309
• Pt. with DM, positive for microalbuminuria should be started on ACEi or
angiotensin-2 receptor antagonists (ARA), irrespective of BP. ACEI is reno-
protective and prevents progress in albuminuria. ACEi is first line and ARAs
for ACEi intolerant individuals.
• Microalbuminuria caused by: DM, #BP, minimal change GN
• Target BP in DM pts
- if no proteinuria: 130/80mmHg
- If microalbuminuria/proteinuria: 125/75mmHg

NOTE:
Statin: cholesterol is normal for this pt. <5.0mmol/L
Amylodipine: CCB, only used for HTN
Biguanide: antihyperglycemic


779. A 32yo female has a hx of SOB and fever. Pre-broncho-dilation test was done
and it was 2/3.5 and post-bronchodilator was 3/3.7. The pt was dx of eczema and TB.
What is the possible dx?
a. COPD
b. Asthma
c. Pneumonia
d. Bronchiectasis

36
RESPIRATORY-System Wise 1700-by Sush and Team. 2016
Susmita, Asad, Manu, Saima, Zohaib, Savia, Shanu, Mona, Manisha, Sitara, Samreena, Sami and Komal


Answer: (b) Asthma: OHCM pg: 172
• S/S: SOB, Intermittent dyspnoea, Wheeze, Cough (often nocturnal),
Tacypnoea, hyperinflated chest
• Precipitants: cold, exercise, NSAIDs, β-blockers, smoking, infection, allergens
• Associated with acid reflux and other atopic diseases: Eczema, hay fever,
allergy, family hx.
• Dx: Pulmonary function tests (PFTs) show an obstructive pattern, FEV1 is
reduced more than the FVC, $FEV1/FVC ratio < 75% (Normal ratio: 75 –
80%); usually > 15% improvement in FEV1 following a bronchodilators as in
this scenario.


307. A 46yo man is being treated for a pleural effusion. A chest drain has been sited
just below the
4th rib in the mid-axillary line on his right side. What single structure is at particular
risk of injury?
a. Arzygos vein
b. Diaphragm
c. Intercostal artery
d. Internal thoracic artery
e. Liver
Ans. The key is C. Intercostal artery. [Most vulnerable structure is intercostal nerve,
then intercostal artery then intercostals vein].

Chest drain is placed between 4th to 6th intercostal space anterior to the mid axillary
line. This is called safe triangle. Sometimes below 7th intercostal space when there is
loculated pleural effusion, or occasionally 2nd intercostal space in the mid clavicular
line for apical pneumothorax.
Injury to neurovascular bundle is a complication. You have to insert the needle
above the 6th rib inorder to prevent injury to 5th intercostal space neurovascular
bundle.

Pleural effusion is fluid in pelural space. Divided by their protein conc. Into
TRANSUDDATES (<25g/L)
EXUDATES(>25g/L)

Blood….haemothorax
Pus….empyema
Chyle (lymph with fat)….chylothorax.


CAUSES..
TRANSUDATES maybe due to increased venous prerssure (cardiac failure, fluid
overload, constrictive pericarditis) OR hypoproteinemia (cirrhosis, nephrotic
syndrome, malapsorption)

37
RESPIRATORY-System Wise 1700-by Sush and Team. 2016
Susmita, Asad, Manu, Saima, Zohaib, Savia, Shanu, Mona, Manisha, Sitara, Samreena, Sami and Komal


EXUDATES…. Due to increased leakiness of pleural capillaires secondary to infection,
infalammation or malignancy… pneumonia, TB, RA, SLE, malignant mets.


SYMPTOMS… asymotiomatic – or dyspnea, pleuritic chest pain.

SIGNS…
Decreased expansion
Stony dull percussion
Diminished breath sounds
Tactile vocal fremitus and vocal resonance decreased.
Tracheal deviation away from effusion.
Signs of associated disease or aspiration marks



TESTS
CXR..blunt costophrenic angles
USG

DIAGNOSTGIC ASPIRATION… send fluid for clinical cghemisatry, bacteriology,
cytology, immunology.


PLEURAL BIOPSY…If fluid analysis inconclusive


MANAGEMENT….is of the underlying cause
DRAINAGE
Pleurodesis
Surgery…persistent collection and increasing pleural thickness on USG require
surgery






335. A 48yo woman who has been taking medications for asthma for a long time has
now presented
with decreasing vision. What is the most probable cause for her decrease in vision?
a. Inhaled salbutamol

38
RESPIRATORY-System Wise 1700-by Sush and Team. 2016
Susmita, Asad, Manu, Saima, Zohaib, Savia, Shanu, Mona, Manisha, Sitara, Samreena, Sami and Komal

b. Inhaled steroids
c. Aminophylline
d. Beta-blockers
e. Oral steroids

Q. 1. What is the key?
Q. 2. Justify the key.

Ans. 1. The key is E. Oral steroid.

Ans. 2. Prolonged steroid use leads to cataract formation, glaucoma and eye
diseases.

Inhaled salbutamol has side effects of tremors, anxiety, decreased K.

Inhaled steroids can cause oral candidiasis.

Aminophylline can cause GI upsets, arrhythmias or fits.

Beta blockers can cause bronchoconstriction etc.

So prolonged use of oral steroids is the correct answser.

Ohcm page 371,,,side effects of steroid use.

ENT.345. Pt presented with hemoptysis 7d post-tonsillectomy. What is the next


step?
a. Packing
b. Oral antibiotics + discharge
c. Admit + IV antibiotics
d. Return to theatre and explore
e. Ice cream and cold fluids

Ans. The key is C. Admit + IV antibiotic. [infection is a common cause of secondary
haemorrhage, haemoorrhage occurring ].

377. A 70yo man admits to asbestos exposure 20yrs ago and has attempted to quit
smoking. He has noted weight loss and hoarseness of voice. Choose the single most
likely type of cancer a.w risk
factors present.
a. Basal cell carcinoma

39
RESPIRATORY-System Wise 1700-by Sush and Team. 2016
Susmita, Asad, Manu, Saima, Zohaib, Savia, Shanu, Mona, Manisha, Sitara, Samreena, Sami and Komal

b. Bronchial carcinoma
c. Esophageal carcinoma
d. Nasopharyngeal carcinoma
e. Oral carcinoma

B. Bronchial carcinoma.
[Asbestos exposure is a risk factor for lung cancer and also has a synergistic effect
with cigarette smoke].

#. Conditions related to asbestos exposure:

1)Pleural plaques (after a latent period of 20-40 yrs)

ii) Pleural thickening
iii) Asbestosis (latent period is typically 15-30 yrs)
iv) Mesothelioma (prognosis is very poor)
v) Lung cancer.

387. A 54yo pt 7 days after a total hip replacement presents with acute onset
breathlessness and raised JVP. Which of the following inv will be most helpful in
leading to a dx?
a. CXR
b. CTPA
c. V/Q scan
d. D-Dimer
e. Doppler US of legs

The key is B. CTPA.



The patient has a +ve two level PE Wells score (if it was negative we should do D-
Dimer) and there is no renal impairment or history suggestive of allergy to contrast
media (if these present we should have go for VQ scan) the investigation of choice is
PTCA. NICE guideline.

According to ohcm, the question of V/Q SCAN or CTPA being a better choice of
investigation in PE, has no clear answer. CTPA is sensitive and specific (>90%). New
multidetector CT systems give faster scanning time and good subsegmental imaging,
requiring less IV contrast, beneficial for patients with renal and cardiac impairment.

40
RESPIRATORY-System Wise 1700-by Sush and Team. 2016
Susmita, Asad, Manu, Saima, Zohaib, Savia, Shanu, Mona, Manisha, Sitara, Samreena, Sami and Komal

1st line is CTPA (which can show clots down to the 5th order pulmonary arteries) and
venous phase CT of the leg veins and pelvis.
CTPA’s advantages are that it is more readily available and that it can show other
lesions.

Bilateral leg ultsounds or venograms may also be sufficient to CONFIRM, but not
exclude, a PE in pateints with a co existing clinical DVT.





924. A 33yo man has a temp=38.5C, cough and chest pain on the right si e on
inspiration. He also has purulent sputum. What is the most likely organism to cause
pneumonia in this pt?
a. Gram +ve diplococci
b. Coagulase +ve cocci
c. PCP cold agglutinins
d. AFB
e. Gram –ve diplococci

Key : c ....... corrected : a
Clincher : no specific pointers here

The most common cause of CAP strept.pneumonia which is a gram +ve diplococci.
Cough with chest pain is a hallmark of CAP.




929. A 65yo man presents with significant weight loss and complains of cough, SOB
and chest pain. Exam: left pupil constricted, drooping of left eyelid. What is the most
likely dx?
a. Pancoast tumor
b. Thoracic outlet syndrome
c. Cervical rib
d. Pneumonia
e. Bronchogenic ca

a. Pancoast tumor
Classically caused by an apical (superior pulmonary sulcus) malignant neoplasm of
the lung. The neoplasm is usually bronchogenic in origin (most commonly squamous
cell carcinoma, sometimes adenocarcinoma and large-cell carcinoma).
• An ipsilateral invasion of the cervical sympathetic plexus leading to Horner's
syndrome (miosis, enophthalmos, ptosis; in 14-50% of patients).
• Ipsilateral reflex sympathetic dystrophy may occur.

41
RESPIRATORY-System Wise 1700-by Sush and Team. 2016
Susmita, Asad, Manu, Saima, Zohaib, Savia, Shanu, Mona, Manisha, Sitara, Samreena, Sami and Komal

• Shoulder and arm pain (brachial plexus invasion C8-T2) leading to wasting of
the intrinsic hand muscles and paraesthesiae in the medial side of the arm.
• Less commonly, unilateral recurrent laryngeal nerve palsy producing
unilateral vocal cord paralysis (hoarse voice ± bovine cough), and/or phrenic
nerve involvement.
• There may be arm oedema secondary to the compression of blood vessels.






935. A 62yo man has been smoking about 15 cigarettes/day for 45yrs, and has been
working as a
builder since he was 24yo. He presents with chest pain, SOB, weight loss. CXR shows
bilateral
fibrosis and left side pleural effusion. What is the best inv that will lead to dx?
a. CXR
b. Pleural fluid aspiration of cytology
c. MRI
d. Pleural biopsy
e. CT

d. Pleural biopsy
Only biopsy confirms carcinoma
Ct is the next step not the best step towards the diagnosis. The best work up leading
to diagnosis should b pleural biopsy,
Asbestosis predisposing to mesothelioma and therefore pleural biopsy
Pleural fluid: straw coloured or blood stained. Cytological analysis occasionally leads
to the diagnosis but a pleural biopsy is usually required.








936. During a basketball match, one of the players suddenly collapsed to the ground
with coughing and SOB. What is the inv of choice?
a. CXR
b. CT
c. MRI
d. V/Q scan
e. CTPA

a. CXR

42
RESPIRATORY-System Wise 1700-by Sush and Team. 2016
Susmita, Asad, Manu, Saima, Zohaib, Savia, Shanu, Mona, Manisha, Sitara, Samreena, Sami and Komal

case of spontaneous pneumothorax...xray chest ...first
Basketball player... tall height.... more chances of apical subpleural blebs... its
SPONTANEOUS PNEUMOTHORAX.. so CXR is best



963. A 73yo man who was a smoker has quit smoking for the past 3yrs. He now
presents with hoarseness of voice and cough since past 3wks. XR: mass is visible in
the mediastinum. What is the best inv to confirm the dx?
a. Bronchoscopy
b. Thoracoscopy
c. US
d. CT thorax
e. LN biopsy

Ans: E
Reason : the patient seems to be affecting from Bronchogenic Carcinoma, The Key
diagnosis as rccomended by NICE
Xray
CT
Bronchoscopy with Biopsy
Biopsy of a convenient metastatic site should be performed if this is easier than
biopsy of a primary site . (Biopsy remains the best investigation to confirm the
diagnosis)
NICE strongly recommends a new imaging test,18F-deoxyglucose positron emission
tomography (FDG-PET), to help stage tumours

Treatment
Small cell tumours are usually treated with 4-6 cycles of multi-drug platinum-based
chemotherapy with the possibility of added radiotherapy in limited stage disease.
Most Rapidly Growing+Worst Prognosis

Staging of the disease and the patient’s overall fitness and exercise ability will
determine the type of treatment chosen
Surgery , Radiotherapy, Radical Radiotherapy , RADIO+ Chemo

883 A 29yo man with hx of asthma comes with post nasal discharge and
bilateral painless nasal blockage. What is the single most likely dx?
a. Nasal polyp
b. Septal hematoma
c. Septal abscess
d. Atopic rhinitis
e. Allergic rhinitis

Clincher(s) Hx Asthma

43
RESPIRATORY-System Wise 1700-by Sush and Team. 2016
Susmita, Asad, Manu, Saima, Zohaib, Savia, Shanu, Mona, Manisha, Sitara, Samreena, Sami and Komal

B & C Not associated with Asthma.
D & E Has watery discharge but no PND (Post Nasal Drip) *Seasonal – hx of
Asthma negative.
A Nasal Polyps

! swelling in the NOSE, maybe in YELLOWISH, GREY or PINK colour.
! cause not known, but ongoing (CHRONIC) inflammation cause swelling
(OEDEMA)
! Polyps usually affect BOTH NOSTRILS & gradually increase in size =
NOSE BLOCK

SYMPTOMS

a) Post Nasal Drip
b) Blocked feeling in nose

*CONDITIONS that make nose inflamed and Polyps more likely =>
1) ASTHMA
2) CYSTIC FIBROSIS
3) ALLERGY TO ASPIRIN
Additional PND = occurs when excessive MUCOUS is produced by NASAL MUCOSA.
Information The excess MUCOSA accumulates in the THROAT and BACK OF THE NOSE.

INVESTIGATION
An ENT surgeon can usually diagnose nasal polyps based on your
symptoms and on examination of your nose.

Occasionally a CT scan or MRI scan may be needed. These scans may
show more detail about where the polyps are and what effects they might
have had on other parts of the face, sinuses and skull.

TREATMENT
Treatment :
betamethasone nose drops
Steroids tablets
Surgery ( polypectomy / endoscopic sinus surgery)

Reference http://patient.info/health/nasal-polyps-leaflet

OHCS 557

44
RESPIRATORY-System Wise 1700-by Sush and Team. 2016
Susmita, Asad, Manu, Saima, Zohaib, Savia, Shanu, Mona, Manisha, Sitara, Samreena, Sami and Komal

45
RESPIRATORY-System Wise 1700-by Sush and Team. 2016
Susmita, Asad, Manu, Saima, Zohaib, Savia, Shanu, Mona, Manisha, Sitara, Samreena, Sami and Komal

890 A 29yo man was involved in an RTA. He presents with distended neck
veins, clear breath sounds and a trachea which is in the midline. His
RR=34bpm, BP=60/0mmHg. What is the most likely dx?
a. Simple pneumothorax
b. Tension pneumothorax
c. Cardiac tamponade
d. Pericarditis

Clincher(s) RTA, CLEAR BREATH SOUNDS & TRACHEA MIDLINE, HYPOTENSION
A Respiratory distress, distant or absent breath sounds (pg 182)
B Absent breath sounds, Trachea shift away from the affected side. (pg
824)
D Pain & High Grade Fever
C CARDIAC TAMPONADE

Related to TRAUMA or HIV – common in young adults.
Related to MALIGNANCY/CHRONIC KIDNEY INJURY – common in elderly

SIGNS

Distended Neck Veins, Hypotension, Tachypnea, Tachycardia and
Hepatomegaly. Muffled heart sounds. Increased JVP. Cyanosis and
Pulmonary Edema may occur.

INVESTIGATION

Transthoracic Echocardiogram

TREATMENT

Monitor in intensive care unit and procedure of choice
PERICARDIOCENTESIS

Additional
Information
Reference http://patient.info/doctor/cardiac-tamponade

OHCS 722
OHCM 787 (procedure)





46
RESPIRATORY-System Wise 1700-by Sush and Team. 2016
Susmita, Asad, Manu, Saima, Zohaib, Savia, Shanu, Mona, Manisha, Sitara, Samreena, Sami and Komal







900 A 70yo pt presents with cough and SOB. He stopped smoking cigarettes
2yrs ago but has a 50yr smoking hx before quitting. CXR=consolidation and
bilateral hilar lymphadenopathy. What is the best inv for this pt?
a. LN biopsy
b. Pleural fluid cytology
c. CT
d. MRI
e. US

Clincher(s) AGE, LONG SMOKING Hx, SOB, COUGH, CONSOLIDATION AND BILATERAL
HILAR LYMPHADENOPATHY
B Only indicates malignancy (cancer)
C / D Helps in staging (cancer) only / CT > MRI
E
A Done for DEFINITVE DIAGNOSIS (best)

Additional If cancer at an early stage shown on CT scan and if patient is fit to be
Information treated, patient may be required to have another type of scan.
This is called a positron emission tomography-CT (PET-CT) scan. This shows
up areas of active cancer and whether it has spread to the lymph glands in
the chest. If the cancer has spread to these glands, you will be offered a
biopsy


Reference http://patient.info/doctor/lung-cancer-pro

47
RESPIRATORY-System Wise 1700-by Sush and Team. 2016
Susmita, Asad, Manu, Saima, Zohaib, Savia, Shanu, Mona, Manisha, Sitara, Samreena, Sami and Komal


903 A pt presents with progressive dyspnea. He complains of cough, wheeze
and a table spoonful of mucopurulent sputum for the last 18m. Spirometry
has been done. FEV1/FVC=2.3/3.6. After taking salbutamol, the
ratio=2.4/3.7. What is the most likely dx?
a. Chronic bronchitis
b. Asthma
c. Bronchiectasis
d. Lung fibrosis
e. Sarcoidosis

Clincher(s) Table spoonful of MUCOPURULENT SPUTUM, FEV1/FVC ratio <75%
A CHRONIC BRONCHITIS
B Asthma has positive effect after Salbutamol >15%
C Copious amount of sputum (FEV1/FVC <75%)
D Dry cough & FEV1/FVC > 75%
E Dry cough & FEV1/FVC > 75%
Additional Chronic Bronchitis ! a type of COPD characterized by AIRWAY
Information OBSTRUCTION
Reference OHCM Pg 176/177 ! COPD & ASTHMA ! 172/173

FOR ASTHMA


FOR COPD

48
RESPIRATORY-System Wise 1700-by Sush and Team. 2016
Susmita, Asad, Manu, Saima, Zohaib, Savia, Shanu, Mona, Manisha, Sitara, Samreena, Sami and Komal


49
RESPIRATORY-System Wise 1700-by Sush and Team. 2016
Susmita, Asad, Manu, Saima, Zohaib, Savia, Shanu, Mona, Manisha, Sitara, Samreena, Sami and Komal




904 A 62yo man presents with cough, breathlessness and wheeze. 24% O2,
salbutamol and hydrocortisone were given. The symptoms haven’t
improved and so nebulized bronchodilator was repeated and IV
aminophylline was given. ABG: pH=7.31, RR=32. What is the next
appropriate management?

a. Nasal IPPV
b. Intubation and ventilation
c. LABA
d. Toxapram
e. Amoxicillin PO

Clincher(s) No improvement after IV aminophylline, pH & RR
A NASAL IPPV
B Done if Nasal IPPV fails
C
D
E
Additional If there is no response to IV Aminophylline, then next step is NIPPV (non-
Information invasive positive pressure ventilation)
Reference OHCM Pg 823

50
RESPIRATORY-System Wise 1700-by Sush and Team. 2016
Susmita, Asad, Manu, Saima, Zohaib, Savia, Shanu, Mona, Manisha, Sitara, Samreena, Sami and Komal

51
RESPIRATORY-System Wise 1700-by Sush and Team. 2016
Susmita, Asad, Manu, Saima, Zohaib, Savia, Shanu, Mona, Manisha, Sitara, Samreena, Sami and Komal


RESPIRATORY 1700

Sush 909, 910, 911, 916, 922



909. A 19yo man has exercised induced asthma and is using a salbutamol inhaler
as req and beclomethasone 400ug BD. He complains that he has to wake up at
night for his inhaler. What is the single most appropriate tx?
a. Beclo
b. Regular salbutamol and budesonide
c. Sodium cromoglycate
d. Oral steroid
e. Inhaled steroid

Key: c
Clincher : night symptoms of asthma


Sush:
Read asthma and treatment : OHCM: 172, 174

Budesonide is a glucocorticoid steroid used in inhaled formulations for the treatment
of asthma,
Sodium cromoglycate prophylactic, inhaled to prevent asthmatic attacks and
allergic reactions. [This is an alternative to beta 2 agonist] fyi: nedocromil sodium is
also an alternative. These are not as effective as B 2 agonist


Usually a long acting beta agonist is needed to cover night symptoms but its not in
options. Others dont make much sense. In essence we want undisturbed sleep.
Sodium chromoglycate is the next best option.

910. Pt with a long hx of smoking is now suffering from bronchial ca. histology
reveals there are sheets of large polygonal or giant MNC. What is the most likely
dx?
a. Squamous cell ca
b. Small cell ca
c. Adenocarcinoma
d. Large cell ca
e. Oat cell ca> OHCM/Gp note says small=oat??

Key: d
Clincher: histology reveals sheets of large polygonal or giant MNC

Large cell carcinoma (lung)

52
RESPIRATORY-System Wise 1700-by Sush and Team. 2016
Susmita, Asad, Manu, Saima, Zohaib, Savia, Shanu, Mona, Manisha, Sitara, Samreena, Sami and Komal

Large multinucleated cells (MNC)


It is a poorly differentiated malignant epithelial tumor. Consists of sheets of large
polygonal or giant MNC. One histological variation is large call neuroendocrine
carcinoma

Sush: The four major histological types of carcinoma of the lung are
• squamous cell carcinoma (SCC), (35%)
• adenocarcinoma (AD), (27%)
• small (oat) cell carcinoma (SCLC) (20%)- paraneoplastic syndromes
(SIADH,ACTH/cushings, etc)associated with these, aggressive, grows rapidly
and metastasize early
• largecell carcinoma (LCC). (10%)

Clinically the most imp division is between SCLC and non-small cell (NSCLC) [Rx- in
OHCM-170]

Need to know major findings in each of above


911. A 27yo man presents with chest pain and respiratory distress. Exam:
tachycardia, hypotension and neck vein distension. Trachea is deviated to the left
side, breathing sounds on right side are absent and diminished on left side. What is
the next appropriate management?
a. CXR
b. Right side aspiration (16G)
c. Left side aspiration (16G)
d. Right side drain with a small tube (12F)
e. Left side drain with a small tube (12F)

Key: b
Clincher : chest pain, resp. distress, hypotension, neck vein distension, deviated
trachea, absent breath sounds

Its a case of tension pneumothorax

Indications of simple aspiration include : Primary spontaneous pneumothorax (PSP)
( any size) , Small SSP in pts. <50yrs
Needle aspiration (14to16G) is as effective as large bore (>20F) chest drain and is
associated with less hospitalisation and less stay.
Following failed needle aspiration, a small bore (<14F) chest drain insertion is
recommended. (Mx OHCM-780)

53
RESPIRATORY-System Wise 1700-by Sush and Team. 2016
Susmita, Asad, Manu, Saima, Zohaib, Savia, Shanu, Mona, Manisha, Sitara, Samreena, Sami and Komal

916. An 83yo man with longstanding COPD has become progressively breathless
over the last 2yrs. He is on salbutamol, ipratropium, salmetarol, beclomethasone
and theophylline. His FEV1<30%. What is the next appropriate management?
a. Lung transplant
b. Trial of CPAP
c. Trial of non-invasive ventilation
d. Assessment for long term O2 therapy
e. Short course of O2 therapy

Key : d
Clincher : every option has been tried, still has SOB, FEV1 is less than 30%

Mx of COPD (OHCM 177)
Lung transplant has no role in old age

when treatment has failed and patient still has SOB ... long term O2 therapy.
Indications of LTO2T include :
FEV1 less than 30%
polycythemia
cyanosis
periphral edema
raised JVP
O2 saturation less than 92%


922. A 71yo man with a hx of 50yrs of smoking presents with cough, hemoptysis,
dyspnea and chest pain. He also has anorexia and weight loss. The dx of lung
cancer has been stabilized (established?). Which electrolyte abnormality can be
seen?
a. Hyperkalemia
b. Hypocalcemia
c. Hyponatremia
d. Hypernatremia
e. Hypomagnesemia

Key : c
Clinchers are for : lung cancer

[Sush: Electrolyte abnormality occurs due to complications of lung cancer (Read
OHCM 170 for all about Lung tumour). One of the endocrine complications is ectopic
hormone secretion, e.g. SIADH (<Na+ and > ADH) (Pg 678 OHCM) due to Small Cell
tumour. Other endocrine effect of SCT is ACTH (Cushings).

[Fyi, >Ca due to PTH by Squamous cell tumours. ]


Syndrome of inappropriate ADH secretion (SIADH)[8][9]

54
RESPIRATORY-System Wise 1700-by Sush and Team. 2016
Susmita, Asad, Manu, Saima, Zohaib, Savia, Shanu, Mona, Manisha, Sitara, Samreena, Sami and Komal

is characterized by excessive release of antidiuretic hormone from the posterior
pituitary gland or another source. The increase in blood volume (hypervolemia)
often results in dilutional hyponatremia in which the plasma sodium levels are
lowered and total body fluid is increased. It was originally described in people
with small-cell carcinoma of the lung, but it can be caused by a number of underlying
medical conditions. The treatment may consist of fluid intake restriction, various
medicines, and management of the underlying cause.]


Inappropriate ADH secretion from posterior pituitary or from ectopic source despite
low serum osmolality.

Major diagnostic features

Hyponatraemia.Plasma hypo-osmolality proportional to
hyponatraemia.Inappropriately elevated urine osmolality (>100 mOsmol/kg)
commonly > plasma osmolarity.Persistent urine [Na+] >40 mmol/L with normal salt
intake.Euvolaemia.Normal thyroid and adrenal function.Extra features include an
elevated ADH level and low blood uric acid level.

Causes (not exhaustive)

Neurological: tumour, trauma, infection, Guillain-Barré syndrome, multiple sclerosis,
systemic lupus erythematosus, intracranial haemorrhage, sinus thrombosis, AIDS,
porphyria.Pulmonary: lung small-cell cancer, mesothelioma, pneumonia, abscess,
cystic fibrosis, asthma, tuberculosis, positive-pressure ventilation.
Other malignancy: oropharyngeal, stomach, pancreas, leukaemia, lymphoma,
thymoma, and genitourinary tract cancers.
Drugs: chlorpropramide, carbamazepine, selective serotonin reuptake inhibitor
(SSRI) antidepressants, tricyclic antidepressants, lithium, MDMA/ecstasy, tramadol,
haloperidol, vincristine, desmopressin, fluphenazine.
Miscellaneous: idiopathic, hereditary, pain, postoperative, stress, endurance
exercise and marathon runners, dermatomal herpes zoster.




RESPIRATORY QUESTIONS FROM 1700 POOL

1418. A 49yo man comes with hx of cough and SOB. His CD4 count is
measured as 350. CXR shows lobar consolidation. What is the single most
appropriate option?

a. Mycobacterium avium intercellular

b. CMV

55
RESPIRATORY-System Wise 1700-by Sush and Team. 2016
Susmita, Asad, Manu, Saima, Zohaib, Savia, Shanu, Mona, Manisha, Sitara, Samreena, Sami and Komal

c. Streptococcus

d. Toxoplasmosis

e. Pneumocystis jerovici

Clinchers: Cough and lobar consolidation.

Explanation:

Streptococcus: The patient presents with h/o cough, SOB and CXR shows lobar
consolidation. these point towards the diagnosis of pneumonia and
streptococcus is the most common cause of community acquired
pneumonia.

CD4 count of uninfected adolescent who is generally in good health ranges from 500
cells/mm3 to 1200 cells/mm3.

MOST COMMON CAUSE OF LOBAR CONSOLIDATION IS STREPTOCOCCUS


PNEUMONIA

How to exclude other options: (Source Patient.info)

Mycobacterium avid intercellular: Patient must have Low CD4 count(less than 200
cells/mm3) and is associated with AIDS. CXR may show cavitary
changes, nodules and parenchymal involvement, particularly in
middle and upper lobes, and mediastinal lymphadenopathy.

CMV:(herpesvirus family) CMV pneumonia is most often seen in severely


immunocompromised patients(CD4 count less than 50 cells/mm3).
But it also occurs in immunocompetent patients. It is often isolated in
patients with AIDS following bronchoalveolar lavage, though it only
occasionally causes pneumonia. It is more significant however, in
transplant patients and those who are iatrogenically
immunosuppressed when it may cause a fatal pneumonitis.

Toxoplasmosis: It is caused by Toxoplasmosis gondi(intracellular protozoan parasite)


and occur in immunocompromised patients. It causes pneumonitis.

PCP: PCP is the most common opportunistic infection in AIDS which usually
occur when CD4 count <200/mm. CXR is either normal or show
bilateral perihilar interstitial shadowing.

Community acquired pneumonia (CAP) may be caused by the following infectious


agents:

56
RESPIRATORY-System Wise 1700-by Sush and Team. 2016
Susmita, Asad, Manu, Saima, Zohaib, Savia, Shanu, Mona, Manisha, Sitara, Samreena, Sami and Komal

• Streptococcus pneumoniae (accounts for around 80% of cases)
• Haemophilus influenzae
• Staphylococcus aureus: commonly after the 'flu
• atypical pneumonias (e.g. Due to Mycoplasma pneumoniae)
• viruses
Klebsiella pneumoniae is classically in alcoholics

Streptococcus pneumoniae (pneumococcus) is the most common cause of
community-acquired pneumonia

Clinical features
Symptoms: Fever, rigors, malaise, anorexia, dyspnoea, cough, purulent sputum,
haemoptysis, and pleuritic pain.
Signs: Pyrexia, cyanosis, confusion (elderly usually), tachypnoea, tachycardia,
hypotension, signs of consolidation (diminished expansion, dull percussion note,
tactile vocal fremitus/vocal resonance, bronchial breathing), and a pleural rub.

Tests:
CXR: lobar or multilobar infiltrates, cavitation or pleural effusion.
Assess oxygenation: oxygen saturation (ABGs if SaO2 <92% or severe pneumonia)
and BP.
Blood tests: FBC, U&E, LFT, CRP, blood cultures. Sputum for microscopy and culture.
Pleural fluid may be aspirated for culture.
Consider bronchoscopy and bronchoalveolar lavage if patient is
immunocompromised or on
ITU.
Management:
CURB-65 criteria of severe pneumonia(each carries one point):

• Confusion (abbreviated mental test score <= 8/10)


• Urea > 7 mmol/L
• Respiratory rate >= 30 / min
• BP: systolic <= 90 or diastolic <= 60 mmHg

• age >= 65 years


0-1: HOME TREATMENT
2: HOSPITAL TREATMENT
more than 3: SEVERE PNEUMONIA. CONSIDER ITU


57
RESPIRATORY-System Wise 1700-by Sush and Team. 2016
Susmita, Asad, Manu, Saima, Zohaib, Savia, Shanu, Mona, Manisha, Sitara, Samreena, Sami and Komal


EMPIRICAL TREATMENT OF PNEUMONIA:(OHCM PG 161)
COMMUNITY ACQUIRED PNEUMONIA:
• MILD: oral amoxicillin or oral clarithromycin 500mg/12h or doxcyline 200mg
loading then 100mg/day.
• MODERATE: oral amoxicillin + oral clarithromycin or doxycycline.
• SEVERE: intravenous co-amoxiclav + clarithromycin OR cefuroxime +
clarithromycin OR cefotaxime + clarithromycin

58
RESPIRATORY-System Wise 1700-by Sush and Team. 2016
Susmita, Asad, Manu, Saima, Zohaib, Savia, Shanu, Mona, Manisha, Sitara, Samreena, Sami and Komal


1431. A man with chronic cough presents with copious purulent sputum. What is the
single most dx?

a. Bronchitis
b. Bronchiectasis
c. COPD
d. Pneumonia
e. Emphysema

Clinchers: Chronic cough and copious purulent sputum.
Diagnosis: Bronchietasis.

HOW TO EXCLUDE OTHER CAUSES:

COPD it includes chronic bronchitis and emphysema. symptoms include chronic
cough with mucoid or mucopurulent sputum. There is usually h/o smoking in these
patients.
PNEUMONIA usually presents with SOB, cough and fever.

59
RESPIRATORY-System Wise 1700-by Sush and Team. 2016
Susmita, Asad, Manu, Saima, Zohaib, Savia, Shanu, Mona, Manisha, Sitara, Samreena, Sami and Komal

EXPLANTION:

Bronchiectasis is described as permanent dilatation of the airways secondary to
chronic infection or inflammation.
Most common organisms isolated from patients with bronchiectasis:
Haemophilus influenzae (most common), Pseudomonas aeruginosa, Klebsiella spp,
Streptococcus pneumoniae
Causes: post-infective:( tuberculosis, measles, pertussis, pneumonia) cystic fibrosis,
bronchial obstruction (e.g. lung cancer/foreign body), immune deficiency: (selective
IgA, hypogammaglobulinemia), allergic bronchopulmonary aspergillosis (ABPA),
ciliary dyskinetic syndromes: (Kartagener's syndrome, Young's syndrome), yellow
nail syndrome
features: persistent cough, purulent copious sputum,intermittent hemoptysis. finger
clubbing, coarse inspiratory crepitations, wheeze ( all found in other diseases)

Tests: sputum culture. chest xray shows cystic shadows, thickened bronchial walls
(tramline and ring shadows). other investigations according to the causes.

Management:

postural drainage,
chest physiotherapy
antibiotics usually for acute exacerbations
surgery in selected cases (e.g localized disease,to control severe hemoptysis)

1492. A retired ship worker has pleural effusion and pleural thickening on
right side with bilateral lung shadowing. What would you do to improve his
symptoms?

a. Aspiration

b. Chest drain (best/aspiration)- in pleural – not chemotherapy

c. Chemotherapy

d. Diuretic

60
RESPIRATORY-System Wise 1700-by Sush and Team. 2016
Susmita, Asad, Manu, Saima, Zohaib, Savia, Shanu, Mona, Manisha, Sitara, Samreena, Sami and Komal

CLINCHERS: Ship worker and pleural effusion with pleural thickening.

DIAGNOSIS: Malignant mesothelioma due to the past exposure of abestos. Asbestos


is used in the insulation of pipe in the heating system of ships.

EXPLANATION:

Malignant mesothelioma is not curable but the palliative treatment with
chemotherapy( Pemerexed+ cisplatin) can improves the survival of patient. Thus, the
symptoms can be controlled with the use of these drugs.

Malignant mesothelioma is a tumour of mesothelial cells that usually occurs in
the pleura, and rarely in the peritoneum or other organs. It is associated with
occupational
exposure to asbestos,

Clinical features: Chest pain, dyspnoea, weight loss, finger clubbing, recurrent pleural
effusions.
Signs of metastases: lymphadenopathy, hepatomegaly, bone pain/tenderness,
abdominal pain/obstruction (peritoneal malignant mesothelioma).


Tests:
CXR/CT: pleural thickening/effusion. Bloody pleural fluid.

Treatment: Patients are usually offered palliative chemotherapy (Pemetrexed +
cisplatin) and there is also a limited role for surgery and radiotherapy.














61
RESPIRATORY-System Wise 1700-by Sush and Team. 2016
Susmita, Asad, Manu, Saima, Zohaib, Savia, Shanu, Mona, Manisha, Sitara, Samreena, Sami and Komal









1496. A 4yo child comes with a sprain in his foot. Hx reveals that the child has had
recurrent admissions to the hosp due to severe asthma. What is the most
appropriate analgesic?
a. Diclofenac sodium

62
RESPIRATORY-System Wise 1700-by Sush and Team. 2016
Susmita, Asad, Manu, Saima, Zohaib, Savia, Shanu, Mona, Manisha, Sitara, Samreena, Sami and Komal

b. Ibuprofen.
c.Paracetamol
d. Codeine

EXPLANATION:

Paracetamol should only be given in this patient to relieve pain.

How to excluded other options:
Diclofenac sodium and Ibuprofen cause NSAID induced asthma and bronchospasm.
While codeine can cause respiratory depression, therefore this drug is also not
recommended for this patient. The only drug which is safest and less risky in the
asthmatic patient is paracetamol.



























1519. A 4yo child presents with repeated chest infections. He has yellow
discoloration of sclera and the mother gives a hx of diarrhea as well. What is the
single inv most likely to lead to a dx?
a. Sweat chloride test
b. Anti-endomysial antiboides
c. LFT
d. Jejunal biopsy

63
RESPIRATORY-System Wise 1700-by Sush and Team. 2016
Susmita, Asad, Manu, Saima, Zohaib, Savia, Shanu, Mona, Manisha, Sitara, Samreena, Sami and Komal

e. TFT

Clinchers: Repeated chest infections, Yellow discolouration of sclera and diarrhea.
Diagnosis: Cystic fibrosis.
MOST LIKELY INVESTIGATION OF CHOICE: SWEAT CHLORIDE TEST.

HOW TO EXCLUDE OTHER OPTIONS:

ANTI-ENDOMYSIAL ANTIBODIES: confirmatory test for coeliac disease and dermatitis
herpetiformis.

LFT: not a confirmatory test, only show picture of obstructive jaundice.

JEJUNAL BIOPSY: Performed as a diagnostic procedure of a number of
gastrointestinal disease including malabsorption disorders e.g coeliac disease,
whipple disease, lymphoma, giardiasis etc.

TFT: done to evaluate thyroid functions. There is no role of doing it in this case.




EXPLANATION: (REF OHCM PG 166)

One of the commonest life-threatening autosomal recessive conditions (1:2000 live
births) affecting Caucasians. Caused by mutations in the CF transmembrane con-
ductance regulator (CFTR) gene on chromosome 7 (>800 mutations have now been
identified). This is a Cl- channel, and the defect leads to a combination of defective
chloride secretion and increased sodium absorption across airway epithelium. The
changes in the composition of airway surface liquid predispose the lung to chronic
pulmonary infections and bronchiectasis.
Features of cystic fibrosis:

High sodium sweat


Primary secretion of sweat duct is normal but CFTR does not absorb chloride ions,
which remain in the lumen and prevent sodium absorption.

Pancreatic insufficiency
Production of pancreatic enzymes is normal but defects in ion transport produce
relative dehydration of pancreatic secretions, causing their stagnation in the
pancreatic ducts.

Biliary disease
Defective ion transfer across the bile duct causes reduced movement of water in the
lumen so that bile becomes concentrated, causing plugging and local damage.

64
RESPIRATORY-System Wise 1700-by Sush and Team. 2016
Susmita, Asad, Manu, Saima, Zohaib, Savia, Shanu, Mona, Manisha, Sitara, Samreena, Sami and Komal

Gastrointestinal disease
Low-volume secretions of increased viscosity, changes in fluid movement across
both the small and large intestine and dehydrated biliary and pancreatic secretions
cause intraluminal water deficiency.

Respiratory disease
Dehydration of the airway surfaces reduces mucociliary clearance and favours
bacterial colonisation, local bacterial defences are impaired by local salt
concentrations and bacterial adherence is increased by changes in cell surface
glycoproteins. Increased bacterial colonisation and reduced clearance produce
inflammatory lung damage due to an exuberant neutrophilic response involving
mediators such as IL8 and neutrophil elastase.
1434,1435,1436,1438,1441 by Manisha on respiratory

1434. A 22yo man reports a 2d hx of hoarseness of voice. He denies any
weight loss but he has been smoking for 4yrs. What is the single most
appropriate inv?
a. None
b. Laryngoscopy
c. Bronchoscopy
d. BAL bronchoalveolar lavage
e. CXR
Answer: A. None. Young patient, only presents with a 2 day h/o
hoarseness of voice and denies weight loss so CA is less likely and
there is no need to investigate.

HOARSENESS OF VOICE
Hoarseness entails difficulty producing sound with change in voice
pitch or quality (‘breathy’,
‘scratchy’, ‘husky’).
Investigate hoarseness (esp in smokers) lasting >3wks, as it is the chief (and
often the only) presentation of laryngeal carcinoma.

Ask about:
Gastro–oesophageal reflux (GORD), dysphagia, smoking, stress, singing
& shouting. Voice overuse is a common cause (prevalence >50% in eg in
teachers).

Tests:
Laryngoscopy (to assess cord mobility, inspect the mucosa, exclude
local causes);
slow motion videolaryngostroboscopy/acoustic analysis
(causes have characteristic vibration patterns).


1435 question :
A 34yo IVDA presents with a 4m hx of productive cough. He has lost

65
RESPIRATORY-System Wise 1700-by Sush and Team. 2016
Susmita, Asad, Manu, Saima, Zohaib, Savia, Shanu, Mona, Manisha, Sitara, Samreena, Sami and Komal

10kgs. What is the single
most appropriate inv?
a. Sputum for AFB: Acid Fast Bacilli
b. Laryngoscopyl
c. Bronchoscopy
d. CT neck
e. CXR
Answer= A. sputum for AFB. (Manisha: the next best step if AFB is not applicable ie
non-productive cough→ do bronchoalveolar lavage) chronic productive cough and
h/o weight
loss points towards the diagnosis of tuberculosis so sputum for AFB
should be done.

Tuberculosis (TB) is an infection caused by Mycobacterium tuberculosis
that most commonly affects the lungs.

primary tuberculosis=
A nonimmune host who is exposed to M. tuberculosis may develop primary
infection of the lungs. A small lung lesion known as a Ghon focus
develops. The Ghon focus is composed of tubercle-laden macrophages.
The combination of a Ghon focus and hilar lymph nodes is known as a
Ghon complex.
In immunocompetent people the initially lesion usually heals by
fibrosis. Those who are immunocompromised may develop disseminated
disease (miliary tuberculosis).
secondary tuberculosis=
If the host becomes immunocompromised the initial infection may become
reactivated. Reactivation generally occurs in the apex of the lungs
and may spread locally or to more distant sites. Possible causes of
immuno compromise include: immunosuppressive drugs like steroids, Hiv,
malignancy.
The lungs remain the most common site for secondary tuberculosis.
Extrapulmonary infection may occur in the following areas:
centralnervous system (tuberculous meningitis - the most serious
complication), vertebral bodies (Pott's disease), cervical lymph nodes
(scrofuloderma), renal, gastrointestinal tract.
risk factors= close contacts of TB patient, alcoholics and other drug
abusers, hiv positive and other immunocompromised patients.etc

presentation=
general symptoms: fever, malaise, night sweats, fatigue, weight loss etc

pulmonary Symptoms= include chronic, productive cough with purulent ±
bloodstained sputum. May result in lobar collapse, bronchiectasis,
pleural effusion and pneumonia.

66
RESPIRATORY-System Wise 1700-by Sush and Team. 2016
Susmita, Asad, Manu, Saima, Zohaib, Savia, Shanu, Mona, Manisha, Sitara, Samreena, Sami and Komal

Nice guideline

Interferon gamma release assay (IGRA)

The interferon gamma release assay (IGRA) is a newer type of blood test for TB that
is becoming more widely available.

The IGRA may be used to help diagnose latent TB:

• if you have a positive Mantoux test


• if you previously had the BCG vaccination (as the Mantoux test may not be
reliable in these cases)
• as part of your TB screening if you have just moved to the UK from a country
where TB is common
• as part of a health check when you register with a GP
• if you are about to have treatment that will suppress your immune system
• if you are a healthcare worker

Screening for latent TB

In some circumstances, you may need to be tested to check for latent TB (when you
have been infected with TB bacteria but do not have any symptoms).

For example, you may need to be screened if you have been in close contact with
someone known to have an active TB infection (an infection that causes symptoms),
or if you have recently spent time in a country where TB levels are high.

If you have just moved to the UK from a country where TB is common, you may be
screened when you arrive or your GP may suggest screening when you register as a
patient




1436. A 25yo pt came to the OPD with complaint of fever, malaise,
breathlessness, cough and anorexia. His gf has got similar symptoms.
He had hx of sore throat and ear discharge a month ago. What
is the single most likely causative organism?
a. Legionella
b. Mycoplasma
c. Chlamydia pneumoniae
d. PCP
e. Chlamydia psittaci
Answer: C.chlamydia pneumoniae. person to person contact, h/o sore
throat and ear discharge are diagnostic points.

67
RESPIRATORY-System Wise 1700-by Sush and Team. 2016
Susmita, Asad, Manu, Saima, Zohaib, Savia, Shanu, Mona, Manisha, Sitara, Samreena, Sami and Komal


Chlamydophila pneumoniae is the commonest chlamydial infection. person
to person spread occurs. features include pharyngitis, hoarseness,
otitis followed by pneumonia.

Diagnosis= chlamydophila complement fixation test,pcr invasive samples

treatment= doxycycline or clarithromycin.
Manisha

Pneumonia
Treatment:
Mild community-acquired Amoxicillin 500mg/8h PO
Possible atypical pneumonia Add erythromycin 500mg/6h PO
Severe community-acquired Co-amoxiclav as above +erythromycin 12.5mg/
kg/6h IV; see legionellosis, p162
Hospital-acquired Ceftazidine (above) or Tazocin® 4.5g/8h IV

✔ C psittaci causes psittacosis or ornithosis after exposure to an
infected bird. Ornithosis is the preferred term, because almost any
bird can transmit the organism. The clinical spectrum of C psittaci
infection ranges from an asymptomatic infection to a fulminant toxic
syndrome. Patients with ornithosis most commonly present with
pneumonia or fever of unknown origin.
✔ pcp occurs in immunocompromised patients
▪ Atypical bacterial pneumonia are as follows
✔ • Psittacosis Q fever Tularemia Mycoplasma pneumonia Legionnaires
disease Chlamydophila ( Chlamydia) pneumonia
▪ Typical bacterial pneumonia:
Staph
Strep
H.influenza
Kliebsella
Pseudomonas
Yersinia
▪ ▪ ▪ ▪ ▪
Atypical Pneumonias
Clinical Features
Radiographic Features
Laboratory Features
Mycoplasma pneumonia
Upper respiratory tract symptoms, Bullous myringitis, Hemolytic
anemia, Myocarditis, Meningoencephalitis
Occasional pleural effusion
Cold hemagglutinins
Legionnaire's disease
Relative bradycardia, Abdominal pain, Vomiting and diarrhea,

68
RESPIRATORY-System Wise 1700-by Sush and Team. 2016
Susmita, Asad, Manu, Saima, Zohaib, Savia, Shanu, Mona, Manisha, Sitara, Samreena, Sami and Komal

Hematuria, Mental confusion
Occasional pleural effusion
Abnormal liver and renal function tests Increased CPK
Psittacosis
Relative bradycardia Epistaxis Horder's spots Splenomegaly

OHCM:
Mycoplasma pneumoniae occurs in epidemics about every 4yrs. It presents
insidiously with fl u-like symptoms (headache, myalgia, arthralgia) followed by a dry
cough.
CXR: reticular-nodular shadowing or patchy consolidation often of 1 lower lobe, and
worse than signs suggest. Diagnosis: PCR sputum or serology. Cold agglutinins may
cause an autoimmune haemolytic anaemia. Complications: skin rash (erythema
multiforme,, Stevens–Johnson syndrome, meningo encephalitis or myelitis;
Guillain–Barre syndrome. Treatment: clarithromycin (500mg/12h) or doxycycline
(200mg loading then 100mg od) or a fl uroquinolone (eg ciprofl oxacin or norfl
oxacin).
Legionella pneumophilia colonizes water tanks kept at <60°C (eg hotel air-
conditioning
and hot water systems) causing outbreaks of Legionnaire’s disease. Flu-like
symptoms (fever, malaise, myalgia) precede a dry cough and dyspnoea. Extra-
pulmonary
features include anorexia, D&V, hepatitis, renal failure, confusion, and coma.
CXR shows bi-basal consolidation. Blood tests may show lymphopenia,
hyponatraemia,
and deranged LFTS. Urinalysis may show haematuria. Diagnosis: Legionella urine
antigen/culture. Treatment: fl uoroquinolone for 2–3wks or clarithromycin (p380).
10% mortality.
Chlamydophila pneumoniae is the commonest chlamydial infection. Person-toperson
spread occurs causing a biphasic illness: pharyngitis, hoarseness, otitis, followed
by pneumonia. Diagnosis: Chlamydophila complement fi xation test, PCR invasive
samples. 8 Treatment: doxycycline or clarithromycin.
Chlamydiophila psittaci causes psittacosis, an ornithosis acquired from infected
birds (typically parrots). Symptoms include headache, fever, dry cough, lethargy,
arthralgia, anorexia, and D&V. Extra-pulmonary features are legion but rare, eg
meningo-encephalitis, infective endocarditis, hepatitis, nephritis, rash, splenomegaly
and chest xray shows patchy consolidation

69
RESPIRATORY-System Wise 1700-by Sush and Team. 2016
Susmita, Asad, Manu, Saima, Zohaib, Savia, Shanu, Mona, Manisha, Sitara, Samreena, Sami and Komal

o


1438. A 50yo DM pt came to the OPD with complaint of of fever, muscle
ache, dry cough and anorexia. Inv: CXR=upper lobe cavitation. What is
the single most likely causative organism?
a. Legionella
b. Mycoplasma
c. Staphylococcus
d. Klebsiella
e. Streptococcus
Answer= D. Klebsiella. elderly, diabetic,upper lobe cavitation all
points towards the diagnosis.

klebsiella pneumonia occurs in elderly, diabetics and alcoholics. flu
like symptoms.It causes a cavitating pneumonia ,particularly of upper
lobes,which is often drug resistant.

treatment= cefotaxime or imipenem
Manisha:
Cavities with a maximum wall thickness of 4 mm or less were usually
[94% of the time]) caused by nonmalignant processes. Cavities with a
maximum wall thickness of 5 to 15 mm were mixed, with (60%) being
nonmalignant and (40%) being malignant cavities. Cavities with a
maximum wall thickness of >15 mm were usually 90% malignant
✔ In the post-antibiotic era pattern of frequency is changing. In
older studies anaerobes were found in up to 90% cases but they are

70
RESPIRATORY-System Wise 1700-by Sush and Team. 2016
Susmita, Asad, Manu, Saima, Zohaib, Savia, Shanu, Mona, Manisha, Sitara, Samreena, Sami and Komal

much less frequent now.
Causing lung cavitation
• Anaerobic bacteria: Actinomyces, Peptostreptococcus, Bacteroides,
Fusobacterium species,
• Microaerophilic streptococcus : Streptococcus milleri
• Aerobic bacteria: Staphylococcus, Klebsiella, Haemophilus,
Pseudomonas, Nocardia, Escherichia coli, Streptococcus,
Mycobacteria[5]
• Fungi: Candida, Aspergillus
• Parasites: Entamoeba histolytica,


1441. A 20yo student who recently visited Asia came to the OPD with
complains of low grade fever,
night sweats, anorexia and productive cough. Inv: CXR=cavitatory
lesions in upper lobes. What is
the single most likely causative organism?
a. Mycoplasma
b. Klebsiella
c. TB
d. PCP
e. Viral pneumonia
Answer= C. Tuberculosis. travel history, low grade fever with night
sweats, anorexia and productive cough, cavitation on chest x ray all
are features of tuberculosis.

investigations for pulmonary TB:
Chest xray= cavitation, consolidation, fibrosis, calcification.

Microbiological samples: firm diagnosis rests on isolating the
infecting organism, and subsequent sensitivity testing can be used to
guide antibiotic therapy.

for respiratory Tb = Send at least three spontaneous sputum samples
for culture and microscopy (including one early morning sample).
Samples are analysed by Staining with Ziehl-Neelsen (ZN) stain and
rapid direct microscopy for acid/alcohol-fast bacilli.
Histologic = hallmark is caseating granulomas
Mantoux test for contact tracing.

Drug treatment for respiratory TB=
The standard therapy for treating active tuberculosis is:

Initial phase - first 2 months (RIPE) 4 2 2 4 (4 drugs for 2 months, then 2 drugs for 4
months)
Rifampicin, Isoniazid, Pyrazinamide, Ethambutol
(the 2006 NICE guidelines now recommend giving a 'fourth drug' such as

71
RESPIRATORY-System Wise 1700-by Sush and Team. 2016
Susmita, Asad, Manu, Saima, Zohaib, Savia, Shanu, Mona, Manisha, Sitara, Samreena, Sami and Komal

ethambutol routinely - previously this was only added if
drug-resistant tuberculosis was suspected)
Continuation phase - next 4 months = Rifampicin,Isoniazid
The treatment for latent tuberculosis is isoniazid alone for 6 months

Patients with meningeal tuberculosis are treated for a prolonged
period (at least 12 months) with the addition of steroids.

Directly observed therapy with a three times a week dosing regimen may be
indicated in certain groups, including: homeless people with active
tuberculosis, patients who are likely to have poor concordance, all
prisoners with active or latent tuberculosis.
Kliebsella ▪ ▪ ▪
✔ most common in hospital acquired
✔ red current jelly sputum
✔ alcoholics
✔ immunocompromised
SE
Rifampicin: alteration of the urine color
Ethambutol: deafness
Sterptomycine: teeth discoloration
Isoniazide: liver abnormalities→ do LFTs
OHCM:
Main side-effects Seek help in renal or hepatic failure, or pregnancy.
Rifampicin: LFT (small AST rise is OK, stop if bilirubin), platelets , orange
discolouration of urine, tears and contact lens, inactivation of the Pill, flu symptoms.
Isoniazid: LFT, WCC , stop if neuropathy and give pyridoxine (50mg/8h PO).
Ethambutol: Optic neuritis (colour vision is the first to deteriorate).
Pyrazinamide: Hepatitis, arthralgia (CI: acute gout; porphyria).


Respiratory Questions 974, 996, 1013, 1062, 1086

Q 974. An elderly man who used to work in the shipyard industry presented with
cough and SOB few weeks to months. He was given salbutamol nebulization and
antibiotics and admitted to the ward. He died 3d later. CT: patchy infiltrates, pleural
thickening and pleural effusion. Why is this a coroner’s case?
u. Pt got wrong dx or management
v. Pt died soon after admission
w. Death could be due to occupational illness



Answer: C

72
RESPIRATORY-System Wise 1700-by Sush and Team. 2016
Susmita, Asad, Manu, Saima, Zohaib, Savia, Shanu, Mona, Manisha, Sitara, Samreena, Sami and Komal

A coroner must hold an inquest if the cause of death is still unknown, or if the
person: possibly died a violent or unnatural death. died in prison or police custody.
person who died was not visited by a medical practitioner during their final illness

medical certificate isn’t available

person who died wasn’t seen by the doctor who signed the medical certificate within
14 days before death or after they died

death occurred during an operation or before the person came out of anaesthetic

medical certificate suggests the death may have been caused by an industrial disease
or industrial poisoning.


Mesothelioma
Mesothelioma is a type of cancer that most often starts in the covering of the lungs
(pleural mesothelioma). It is associated with occupational exposure to asbestos . The
latent period between exposure and development of disease could be as long as 45
years .

Asbestos is used widely in the

• Building industry
• Ship building industry
• Manufacture of household appliances
• Motor industry

The symptoms of pleural mesothelioma

The symptoms of pleural mesothelioma are

• Chest pain
• Shortness of breath
• Tiredness (fatigue)
• Sweating and high temperatures
• A persistent cough
• Losing weight when not dieting
• Loss of appetite
• Difficulty swallowing
• A hoarse or husky voice

Some people have changes in the shape of their fingers and nails called finger
clubbing.

73
RESPIRATORY-System Wise 1700-by Sush and Team. 2016
Susmita, Asad, Manu, Saima, Zohaib, Savia, Shanu, Mona, Manisha, Sitara, Samreena, Sami and Komal


Many people who develop mesothelioma due to asbestos exposure may be able to
claim compensation. It is important to talk to a solicitor about this as early as
possible. Your specialist doctor or nurse may be able to give you some information.
Or some of the mesothelioma organisations can help and advise you.

Q 996. A 64yo man complains of increasing SOB and cough for the past 18m. He
coughs up a Tbsp of mucopurulent sputum with occasional specks of blood. What is
the most likely underlying cause?
a. Acute bronchitis
b. Bronchiectasis
c. Chronic bronchitis
d. Lung cancer
e. Pneumonia



Bronchiachtasis and Chronic Bronchitis

Bronchitis is inflammation of the bronchi (large and medium-sized
airways) of the lungs.[1] Symptoms include coughing up mucus,
wheezing, shortness of breath, and chest discomfort .
Acute bronchitis usually has a cough that lasts around three weeks.[2]
In more than 90% of cases the cause is a viral infection
Chronic bronchitis is defined clinically as cough and sputum
production on most days for three months of 2 successive years.
Brochiectasis is a disease in which there is permanent enlargement
of parts of the airways of the lung.[1] Symptoms typically include a
chronic cough productive of mucus.[2] Other symptoms include
shortness of breath, coughing up blood, and chest pain.[1] Wheezing
and nail clubbing may also occur. Those with the disease often get
frequent lung infections


Q 1013. An 89yo pt has lung cancer. His Na+=122mmol/l. What is the tx for
this?
x. Demeclocycline
y. Vasopressin
z. Restrict fluids
aa. Reassure


Demeclocycline or Restrict fluids

Hyponatremia

74
RESPIRATORY-System Wise 1700-by Sush and Team. 2016
Susmita, Asad, Manu, Saima, Zohaib, Savia, Shanu, Mona, Manisha, Sitara, Samreena, Sami and Komal

Hyponatremia is defined as a serum sodium level of less than 135
mEq/L and is considered severe when the serum level is below 125
mEq/L.
Plasma Na+ concentration depends on amount of both Na+ and water
in the plasma. Hyponatremia therefore doesn’t necessarily imply
Na+ depletion.
Causes table on page 687 ohcm
S/s
Look for anorexia , nausea and malaise initially followed by headache,
of irritability, confusion, weakness low GCS (Glasgow coma scale) and
seizures depending on severity and rate of change in serum NA+.



Management
% Correct the underlying cause never base treatment on Na+
concentration alone
% Presence of symptom , chronicity of hyponatremia and state of
hydration are all important
% Replace Na+ and water at the same rate they are lost
% See page 687 ohcm


Q 1062. A nonsmoker who has worked in coal mines for 20yrs presents with
gradually increasing SOB, limited exercise tolerance and a dry cough. His CXR
shows round fibrotic tissue demonstrating a mixed restrictive and obstructive
ventilator defect with irreversible airflow limitation and reduced gas transfer.
What is the single most appropriate dx?
bb. Churg-strauss syndrome
cc. Cryptogenic organizing
dd. Extrinsic allergic alveolitis
ee. Good pasture’s syndrome
ff. Progressive massive fibrosis
gg. Molluscum

Answer: I
CWP coal workers pneumoconiosis
It results from inhalation of coal dust particles (1-3um in
diameter)over 15-20 years. These are ingested by macrophages which
die releasing their enzymes and causing fibrosis.
Asymptomatic but coexisting chronic bronchitis is common. CXR
many round opacities (1-10mm)especially upper zone.
Management: Avoid exposure to coal dust, treat coexisting chronic
bronchitis
Progressive massive fibrosis

75
RESPIRATORY-System Wise 1700-by Sush and Team. 2016
Susmita, Asad, Manu, Saima, Zohaib, Savia, Shanu, Mona, Manisha, Sitara, Samreena, Sami and Komal

Is due to progression of cwp , which causes progressive dyspnea,
fibrosis and eventually cor pulmonale . Cxr shows upper zone fibrotic
masses (1-10cm)
GoodPasture’s: Glomerulonephritis + Pulmonary symptoms


Q 1086. A 63yo man presented with sudden onset of severe dyspnea,
orthopnea, raised JVP and bilateral basal crackles 3d after an episode of MI. A
dx of acute congestive cardiac failure was made and IV furosemide was
started for this pt. What electrolyte abnormality is expected?
hh. High Na+, Low K+
ii. Low Na+, High K+
jj. Low Na+, Low K+
kk. High Na+, High K+
ll. Low Na+, Normal K+

Answer:P
Furosemide, like other loop diuretics, acts by inhibiting NKCC2, the luminal
Na-K-2Cl symporter in the thick ascending limb of the loop of Henle. The
action on the distal tubules is independent of any inhibitory effect on
carbonic anhydrase or aldosterone; it also abolishes the corticomedullary
osmotic gradient and blocks negative, as well as positive, free water
clearance.

As with many diuretics, it can cause dehydration and electrolyte
imbalance, including loss of potassium, calcium, sodium, and
magnesium

Respiratory Questions 974, 996, 1013, 1062, 1086

Q 974. An elderly man who used to work in the shipyard industry presented with
cough and SOB few weeks to months. He was given salbutamol nebulization and
antibiotics and admitted to the ward. He died 3d later. CT: patchy infiltrates, pleural
thickening and pleural effusion. Why is this a coroner’s case?
mm. Pt got wrong dx or management
nn. Pt died soon after admission
oo. Death could be due to occupational illness



Answer: C

A coroner must hold an inquest if the cause of death is still unknown, or if the
person: possibly died a violent or unnatural death. died in prison or police custody.
person who died was not visited by a medical practitioner during their final illness

76
RESPIRATORY-System Wise 1700-by Sush and Team. 2016
Susmita, Asad, Manu, Saima, Zohaib, Savia, Shanu, Mona, Manisha, Sitara, Samreena, Sami and Komal

medical certificate isn’t available

person who died wasn’t seen by the doctor who signed the medical certificate within
14 days before death or after they died

death occurred during an operation or before the person came out of anaesthetic

medical certificate suggests the death may have been caused by an industrial disease
or industrial poisoning.


Mesothelioma
Mesothelioma is a type of cancer that most often starts in the covering of the lungs
(pleural mesothelioma). It is associated with occupational exposure to asbestos . The
latent period between exposure and development of disease could be as long as 45
years .

Asbestos is used widely in the

• Building industry
• Ship building industry
• Manufacture of household appliances
• Motor industry

The symptoms of pleural mesothelioma

The symptoms of pleural mesothelioma are

• Chest pain
• Shortness of breath
• Tiredness (fatigue)
• Sweating and high temperatures
• A persistent cough
• Losing weight when not dieting
• Loss of appetite
• Difficulty swallowing
• A hoarse or husky voice

Some people have changes in the shape of their fingers and nails called finger
clubbing.

77
RESPIRATORY-System Wise 1700-by Sush and Team. 2016
Susmita, Asad, Manu, Saima, Zohaib, Savia, Shanu, Mona, Manisha, Sitara, Samreena, Sami and Komal

Many people who develop mesothelioma due to asbestos exposure may be able to
claim compensation. It is important to talk to a solicitor about this as early as
possible. Your specialist doctor or nurse may be able to give you some information.
Or some of the mesothelioma organisations can help and advise you.

Q 996. A 64yo man complains of increasing SOB and cough for the past 18m. He
coughs up a Tbsp of mucopurulent sputum with occasional specks of blood. What is
the most likely underlying cause?
f. Acute bronchitis
g. Bronchiectasis
h. Chronic bronchitis
i. Lung cancer
j. Pneumonia



Bronchiachtasis and Chronic Bronchitis

Bronchitis is inflammation of the bronchi (large and medium-sized
airways) of the lungs.[1] Symptoms include coughing up mucus,
wheezing, shortness of breath, and chest discomfort .
Acute bronchitis usually has a cough that lasts around three weeks.[2]
In more than 90% of cases the cause is a viral infection
Chronic bronchitis is defined clinically as cough and sputum
production on most days for three months of 2 successive years.
Brochiectasis is a disease in which there is permanent enlargement
of parts of the airways of the lung.[1] Symptoms typically include a
chronic cough productive of mucus.[2] Other symptoms include
shortness of breath, coughing up blood, and chest pain.[1] Wheezing
and nail clubbing may also occur. Those with the disease often get
frequent lung infections


Q 1013. An 89yo pt has lung cancer. His Na+=122mmol/l. What is the tx for
this?
pp. Demeclocycline
qq. Vasopressin
rr. Restrict fluids
ss. Reassure


Demeclocycline or Restrict fluids

Hyponatremia
Hyponatremia is defined as a serum sodium level of less than 135
mEq/L and is considered severe when the serum level is below 125
mEq/L.

78
RESPIRATORY-System Wise 1700-by Sush and Team. 2016
Susmita, Asad, Manu, Saima, Zohaib, Savia, Shanu, Mona, Manisha, Sitara, Samreena, Sami and Komal

Plasma Na+ concentration depends on amount of both Na+ and water
in the plasma. Hyponatremia therefore doesn’t necessarily imply
Na+ depletion.
Causes table on page 687 ohcm
S/s
Look for anorexia , nausea and malaise initially followed by headache,
of irritability, confusion, weakness low GCS and seizures depending on
severity and rate of change in serum NA+.



Management
% Correct the underlying cause never base treatment on Na+
concentration alone
% Presence of symptom , chronicity of hyponatremia and state of
hydration are all important
% Replace Na+ and water at the same rate they are lost
% See page 687 ohcm


Q 1062. A nonsmoker who has worked in coal mines for 20yrs presents with
gradually increasing SOB, limited exercise tolerance and a dry cough. His CXR
shows round fibrotic tissue demonstrating a mixed restrictive and obstructive
ventilator defect with irreversible airflow limitation and reduced gas transfer.
What is the single most appropriate dx?
tt. Churg-strauss syndrome
uu. Cryptogenic organizing
vv. Extrinsic allergic alveolitis
ww. Good pasture’s syndrome
xx. Progressive massive fibrosis
yy. Molluscum

Answer: I
CWP coal workers pneumoconiosis
It results from inhalation of coal dust particles (1-3um in
diameter)over 15-20 years. These are ingested by macrophages which
die releasing their enzymes and causing fibrosis.
Asymptomatic but coexisting chronic bronchitis is common. CXR
many round opacities (1-10mm)especially upper zone.
Management: Avoid exposure to coal dust, treat coexisting chronic
bronchitis
Progressive massive fibrosis
Is due to progression of cwp , which causes progressive dyspnea,
fibrosis and eventually cor pulmonale . Cxr shows upper zone fibrotic
masses (1-10cm)

79
RESPIRATORY-System Wise 1700-by Sush and Team. 2016
Susmita, Asad, Manu, Saima, Zohaib, Savia, Shanu, Mona, Manisha, Sitara, Samreena, Sami and Komal

Q 1086. A 63yo man presented with sudden onset of severe dyspnea,
orthopnea, raised JVP and bilateral basal crackles 3d after an episode of MI. A
dx of acute congestive cardiac failure was made and IV furosemide was
started for this pt. What electrolyte abnormality is expected?
zz. High Na+, Low K+
aaa. Low Na+, High K+
bbb. Low Na+, Low K+
ccc. High Na+, High K+
ddd. Low Na+, Normal K+

Answer:P
Furosemide, like other loop diuretics, acts by inhibiting NKCC2, the luminal
Na-K-2Cl symporter in the thick ascending limb of the loop of Henle. The
action on the distal tubules is independent of any inhibitory effect on
carbonic anhydrase or aldosterone; it also abolishes the corticomedullary
osmotic gradient and blocks negative, as well as positive, free water
clearance.

As with many diuretics, it can cause dehydration and electrolyte
imbalance, including loss of potassium, calcium, sodium, and
magnesium

Qs 1240,1243,1303,1308 and 1332 Saima


1240. A 55yo male presents to the ED after an RTA with breathlessness, engorged
neck veins and a
dull percussion note on the right side of his chest. Exam: pulse=140 bpm, BP=80/50
mmHg. What
is the most likely dx?
a. Hemothorax
b. Hemopneumothorax
c. Tension pneumothorax
d. Simple pneumothorax

Hemopneumothorax (B)
Hemothorax is rarely a solitary finding in blunt trauma.
Dull percussion note, in particular, takes the diagnosis away from any kind of
pneumothorax.
Low BP in this scenario indicates shock secondary to heavy blood loss
The reason Hemopneumothorax is the correct option is the presence of engorged
neck veins. Looking at the stem of the question now, we can see that the patient has
signs of both Hemothorax and Pneumothorax. Therefore the correct choice is B.

Saima

80
RESPIRATORY-System Wise 1700-by Sush and Team. 2016
Susmita, Asad, Manu, Saima, Zohaib, Savia, Shanu, Mona, Manisha, Sitara, Samreena, Sami and Komal

Although position of trachea is not mentioned in this question but clincher such as “
engorged veins” shows the tension pneumothorax.



1243. A 30yo woman is taking tx for asthma. She has a HR=130bpm and peak
expiratory flow rate=400.
What is the most appropriate management?
a. Atenolol
b. Digoxin
c. Review drugs

Asthma (C)
The patient has a heart rate that is more than a 110, which means that she is having
a severe attack of asthma. Her expiratory flow rate is also decreased. She is already
on medication which apparently is not adequate to alleviate her symptoms. Digoxin
and Atenolol have no role in the treatment of Asthma. The correct option is to
review the drugs she is taking.
- Treatment of severe or life-threatening asthma
- Salbutamol 5m nebulized with oxygen and give prednisolone 30 mg PO
- If PEF remains less than 75%, repeat Salbutamol
- Monitor O2 saturation, HR and RR.

Saima
Normal value of peak expiration flow rate for man is > 500ml/min
For woman its 420ml/min
Page 173 OHCM
Heart rate above 110/ min shows deterioration of situation in asthma along with low
PEF.


1303. A 62yo prv shipyard worker complains of breathlessness and chest pain for
6m. He has now developed a large pleural effusion. Which is the single best
diagnostic inv?
a. ABG
b. Bronchoscopy
c. CXR
d. Pleural biopsy
e. Transfer factor

ans. Pleural biopsy
Dx Malignant mesothelioma
Malignant mesothelioma should be considered in patients with history of asbestos
exposure and presenting with pleural effusion+chest pain .
Mesothelioma is an uncommon type of cancer that occurs in the tissues covering the
lungs or the abdomen. Past exposure to asbestos is a risk factor for mesothelioma.

81
RESPIRATORY-System Wise 1700-by Sush and Team. 2016
Susmita, Asad, Manu, Saima, Zohaib, Savia, Shanu, Mona, Manisha, Sitara, Samreena, Sami and Komal

Initial symptoms are variable but can include shortness of breath, chest pain or
abdominal swelling, cough, hoarse




1308 . A 24yo male is admitted with acute severe asthma. Tx is initiated with 100%
oxygen, nebulized
salbutamol and ipratropium bromide nebulizers and IV hydrocortisone. Despite
initial tx there is
no improvement. Which is the next step in management?
a. IV aminophylline
b. IV magnesium sulphate
c. IV salbutamol
d. IM adrenaline
e. IV adrenaline
Ans: B IV magnesium sulphate
British Thoracic Society guidelines:
magnesium sulphate recommended as next step for patients who are not
responding (e.g. 1.2 - 2g IV over 20 mins)
little evidence to support use of IV aminophylline (although still mentioned in
management plans)
if no response consider IV salbutamol.
Medscape: Magnesium sulfate has been shown to inhibit smooth muscle
contraction, decrease histamine release from mast cells, and inhibit acetylcholine
release

1332. A 5yo boy is rescued from a burning building and is presented to the ED. He
has 5% partial
thickness burns over the arms and legs and had soot in the mouth and nose. His
breathing has
become noisy. What is the single most immediate management?
a. Nebulized adrenaline
b. Nebulized salmetarol and oxygen
c. Needle cricothyrodotomy
d. Oropharyngeal airway
e. Intubation of airway
Ans : E
Intubation of airway should be done immediately in order to prevent blockage of
airways secondary to laryngeal edema.
Always remember in emergency follow ABCDE which is airway, breathing,
circulation, disability and exposure


Sami
1379. A man suffering from Influenza A since 5d ago. CXR: pneumonia. What
organism is responsible

82
RESPIRATORY-System Wise 1700-by Sush and Team. 2016
Susmita, Asad, Manu, Saima, Zohaib, Savia, Shanu, Mona, Manisha, Sitara, Samreena, Sami and Komal

for pneumonia in this pt?
a. Hemophilius influenze
b. Klebsiella
c. Staphylococcus aureus
d. Streptococcus pneumonia
e. Pseudomonas

Answer: C.
Clincher: a pneumonia after viral infection → Staph. Aureus
CXR: consolidation, sometimes bilateral basal caviation
Rx: Flucloxacillin
(MRSA→ Vancomycin)

1646. An 8yo girl is complying with her asthma treatment of low-dose inhaled
corticosteroid
prophylaxis and short-acting bronchodilators as required. Her inhaler technique is
good.
She
now has a frequent night cough and mild exercise-induced wheeze. What would be
the
SINGLE
most appropriate change in her treatment?
a. Add leukotriene antagonist
b. Add oral theophylline
c. Add regular long-acting bronchodilator
d. Increase dose of inhaled corticosteroid
e. Short course of oral corticosteroid

Answer: C
The Key is D which is wrong

83
RESPIRATORY-System Wise 1700-by Sush and Team. 2016
Susmita, Asad, Manu, Saima, Zohaib, Savia, Shanu, Mona, Manisha, Sitara, Samreena, Sami and Komal


Dr RAbia
A preventer inhaler (usually a steroid inhaler), taken each morning and at
bedtime. This usually prevents symptoms throughout the day and night.
· A reliever inhaler (short acting bronchodilators) may be needed now and
then if breakthrough symptoms occur. For example, if symptoms flare up
when you have a cough or cold.
· If exercise or sport causes symptoms then a dose of a reliever inhaler just
before the exercise usually prevents symptoms.
· The dose of the preventer inhaler may need to be increased for a while if
you have a cough or cold, or during the hay fever season
1673. A 20yo prv healthy woman presents with general malaise, severe cough and
breathlessness which has not improved with a seven day course of amoxycillin.
There is nothing
significant to find on examination. The x-ray shows patchy shadowing throughout
the lung fields. The
blood film shows clumping of red cells with suggestion of cold agglutinins.
a. Mycobacterium avium complex
b. Coxiella burnetii
c. Escherichia coli (Gram -ve)
d. Haemophilus influenza
e. Legionella pneumophila
f. Strep pneumococcus
g. TB
h. Mycoplasma pneumonia
i. PCP
j. Staph aureus
Answer: A.

84
RESPIRATORY-System Wise 1700-by Sush and Team. 2016
Susmita, Asad, Manu, Saima, Zohaib, Savia, Shanu, Mona, Manisha, Sitara, Samreena, Sami and Komal

The most important catcher is: “clumping of red cells with suggestion of cold
agglutinins”
Other catcher: young otherwise healthy, not responding to penicillin, no mention for
productive cough, and the PATCHY shadowing in the CXR (NOTICE: Pneumocystis
Jiroveci in CXR: Bilateral perihilar interstitial shadowing)
Rx of mycoplasma: Clarithromycin, or Doxycyline or Fluoroquinolone
1682. A patient with chronic neutropenia develops a chronic cough. A CXR reveals a
cavitating intrapulmonary lesion containing a movable rounded ball lesion. A likely
dx is?
a. Tuberculosis
b. Bronchiectasis
c. Cystic fibrosis
d. Pulmonary hemosiderosis
e. Mitral stenosis
f. Aspergillosis
g. Wegener’s granulomatosis
h. Goodpasture’s syndrome
i. Pulmonary embolism
j. Non-SCLC
A: F.
Clincher: A ball sitting in a cavity in CXR of a patient with
neutropenia/immunodeficiency
OHCM p 168: Aspergillus affects the lung in 5 ways:
1. Asthma
2. Allergic bronchopulmonary aspergillosis ABPA: affects 1% of asthmatics, 2-
25% of CF patients. → bronchiectasis, recurrent pneumonia. Rx Prednisolone
+-itraconazole. Bronchodialator for asthma. Apsiration of mucus plugs is
needed
3. Aspergilloma (mycetoma) A fungus ball within a pre-exiting cavity (often
caused by TB or sarcoidosis)
C/F: usually asymptomatic. May cause caough, haemoptysis (may be
torrential = streaming.
CXR: round opacity within a cavity, usually apical
Rx: only if symptomatic, surgical, itroconazole (limited success), local
instillation of amphotericin under CT guidance for patients with massive
haemoptysis.
4. Invasive apsergillosis: Risk factors: immunocompromise, or after broad-
spectrum antibiotics.
Rx: vorconazole (superior to Amphotericine B
5. Extrinisic Allergic Alveolitis (EEA) sensitivity to Aspergillus Clavatus. Rx:
prednisolone
1683. A mother brings her 1yo infant to her pediatrician. She describes that
following a
common cold
her child's voice has become hoarse and has developed a cough that sounds

85
RESPIRATORY-System Wise 1700-by Sush and Team. 2016
Susmita, Asad, Manu, Saima, Zohaib, Savia, Shanu, Mona, Manisha, Sitara, Samreena, Sami and Komal

harsh and
brassy
and was worse at night. Exam: the child was noted to have trouble drawing air
into its
lungs
between coughs and had trouble drawing air into its lungs. There was visible
stridor on
inhalation. The cause is most likely to be?
a. EBV
b. Rhinovirus
c. Parainfluenza
d. Flavivirus
e. HIV
f. Rotavirus
g. CMV
h. Kemerovo
i. Creutzfeld-Jacob
j. Rubella
Answer: C
OHCS P. 158 & 566
Croup: (acute laryngotracheobronchitis) Signs: stridor, barking cough, hoarseness
from obstruction in the region of layrnics, <6 y old, usually during Autumn. If
there is cough and no drooling, croup is almost always the diagnosis. (exclude
Acute epiglottitis)
Causes 95% viral: Parainfluenza virus, respiratory syncytial virus, measles.,
Bacteria (klebsiella, diphtheria.
Patho: subglottic oedema, inflammation and exudate.
Rx:
Mild-moderate Croup: Dexamethasone or prednisolone
Sever: antibiotics, humified O2 + nebulized adrenaline. Dexamethasone po stat
or budesonide nebulised
DDx Acute epiglottitis: rare but life-threatening: male: femal = 3:1. Sudden
onset, septicaemia is rapid. Cough is absent, drooling (head forward tongue out),
prefers to sit, refusal to wallow, sysphagia. Aslso, sore throate, fever, dyspnea,
voice change, tender ant. Neck.
Cause: haemophilus (vaccination has reduced prevalence in the UK), strep
pyogenes.
Mx: ITU, don’t exaime throught (cause resp arrest). O2, nebulizer adrenaline, IV
Dexamethasone, IV penicillin and ceftriaxone, antipyretic (ibuprofen)

86
RESPIRATORY-System Wise 1700-by Sush and Team. 2016
Susmita, Asad, Manu, Saima, Zohaib, Savia, Shanu, Mona, Manisha, Sitara, Samreena, Sami and Komal


1432, 1560, 1568, 1569, 1696 - Shanu

87
RESPIRATORY-System Wise 1700-by Sush and Team. 2016
Susmita, Asad, Manu, Saima, Zohaib, Savia, Shanu, Mona, Manisha, Sitara, Samreena, Sami and Komal

1432. A 32yo man working in a shipyard comes with SOB. Exam: dullness on left side
of the chest, pain in left side of chest, pleuritic rub and crackles been heard on the
same side. What is the single most likely dx?
a. Pericarditis
b. Pleurisy
c. Pleural effusion
d. CCF
e. TB


Answer= C. Pleural effusion.
Shipyard worker therefore exposure to asbestos which leads to (asbestosis – since
pt. is young) mesothelioma (The latent period between exposure and development
of the tumour may be up to 45yrs) causing pleural effusion. dyspnoea, pleuritic chest
pain, decreased chest expansion, stony dull percussion note, diminished breath
sounds on affected side are its features.

Pleural Effusion is the fluid in the pleural space.
Effusions can be divided by their protein concentration into transudates (<25g/l) and
exudates (>35g/l)
Causes=
Transudates: due to increased venous pressure, commonly heart failure, cirrhosis,
hypoalbuminemia, peritoneal dialysis, constrictive pericarditis, hypothyroidism,
nephrotic syndrome etc
Exudative: pneumonia, malignancy (bronchogenic ca, mesothelioma, lymphoma etc),
TB, rheumatoid arthritis,pulmonary infarction, pancreatitis, SLE etc

Features= asymptomatic or dyspnea, pleuritic chest pain, decreased chest
expansion, stony dull percussion and diminished breath sounds on the affected side.
features of other diseases may be present.
Tests= Chest X-ray: this is the first investigation if a pleural effusion is suspected
clinically. small effusions blunt the costophrenic angles. large are seen as water
dense shadows with concave upper borders. Bilateral effusions with an enlarged
heart shadow are commonly caused by congestive cardiac failure.

Ultrasound= done to identify the presence of fluid and in guiding therapeutic or
diagnostic aspiration.
Diagnostic aspiration and pleural biopsy are also done.

Management:
Small effusions that are not causing respiratory embarrassment may be managed by
observation.
symptomatic effusions= drainage. fluid is best removed slowly (0.5 -1.5L/hr)
Pleurodesis with tetracycline, bleomycin or talc may be helpful for recurrent
effusions.
surgery= persistent collections and increasing pleural thickness (on U/S) require
surgery

88
RESPIRATORY-System Wise 1700-by Sush and Team. 2016
Susmita, Asad, Manu, Saima, Zohaib, Savia, Shanu, Mona, Manisha, Sitara, Samreena, Sami and Komal


Asbestosis CF: Similar to other fibrotic lung diseases with progressive dyspnoea,
clubbing, and fine end-inspiratory crackles. Also causes pleural plaques, risk of
bronchial adenocarcinoma and mesothelioma
Pleural rubs: caused by movement of visceral pleura over parietal pleura, when both
surfaces roughened, eg by inflammatory exudate.

1560. A patient who works in a pet shop has temp = 37.5C, dyspnoea, chest pain and
cough. CXR: patchy consolidation. What is the most suitable t/m?
a. Amoxicillin
b. Tetracycline
c. Erythromycin
d. Clarithromycin
e. Penicillin


Key: Amoxicillin (A) is wrong- The corrected answer is B Tetracycline
Reason: This is a mild case of Community Acquired Pneumonia (CAP) and the patient
has presented to you in a GP setting. Now, while the history would point towards
Chlamydia pneumonia or Chlamydia psittaci being the commoner cause of this
pneumonia, unless that is proved, you will treat this as Streptococcus pneumonia
acquired pneumonia which is the commonest form of pneumonia in this scenario.
Treatment of choice will be Oral Amoxicillin 500mg – 1g x 8 hourly.

Chlamydia Pistacci (OHCM pg: 162)
Hx: contact with birds (typically parrots)
S/S: dry cough, headache, fever, lethargy, arthralgia, anorexia and D&V
Investigation: CXR shows patchy consolidation
Dx of Chlamydia piSttacci = chlamydophilia serology
Tx: Tetracycline or clarithromycin


1568. A 10yo boy has fallen from a tree and injured his right chest. He has pain and
difficulty breathing. He is tachypneic and tender with an area of paradoxical chest
wall movement on the right side. What is the single most likely dx?
a. Diaphragmatic rupture
b. Flail chest
c. Fractured ribs
d. Hemothorax
e. Tension pneumothorax

Key: Flail Chest (B)
Reason: H/O Fall, Pain and difficulty breathing, Tender with paradoxical chest wall
movement ---> FLAIL CHEST.
Discussion: A case of flail chest should be managed according to the following
principles:
* High flow O2.

89
RESPIRATORY-System Wise 1700-by Sush and Team. 2016
Susmita, Asad, Manu, Saima, Zohaib, Savia, Shanu, Mona, Manisha, Sitara, Samreena, Sami and Komal

* CPAP.
* Chest physiotherapy.
* Adequate analgesia.
* Early intubation and ventilation in patients going towards or currently in
respiratory failure.
* Open fixation for patient who are unable to be weaned from ventilator or when
thoracotomy is performed for other injuries.


1569. A 37yo woman had an elective LSCS 1d ago. You are called to see her as she
becomes SOB with left sided chest pain and a cough. She has had 3 children, 2 born
by LSCS. Exam: she has reduced air entry at left lung base. Her observations include
sat=92% on air, BP=105/84mmHg, pulse=120bpm, temp=37.2C. Choose among the
options which C-section complications has she developed?
a. Aspiration pneumonia
b. Aspiration pneumonitis
c. Spontaneous pneumothorax
d. Pulmonary embolism
e. DVT

Key: Pulmonary embolism (D)
Reason: Decreasing saturation, Tachycardia, Presentation 1 day after major surgery
after Pregnancy and bring post-partum. Reduced air entry cannot be positive in any
other option and DVT would actually lead to Pulmonary Embolism. Pulmonary
Embolism is especially common in abdominal, pelvic surgeries ad hip/knee
replacements. Pregnancy and post-partum state is also a risk factor.
Discussion: Usually arises from a venous thrombosis in the pelvis or legs. Rare causes
include right ventricular thrombus, post MI, septic emboli, right sided endocarditis,
fat/air/amniotic fluid embolism, neoplastic cells, parasites.
Risk factors: Recent surgery especially abdominal, pelvic or hip/knee replacement.
Thrombophilia (antiphospholipid syndrome), leg fracture, prolonged bed rest,
malignancy, pregnancy, post-partum state, COCPs, HRT, previous pulmonary
embolism.
Symptoms: Acute dyspnoea, pleuritic chest pain, hemoptysis, dizziness, syncope.
Signs: Pyrexia, cyanosis, tachypnoea, tachycardia, hypertension, raised JVP, pleural
rub, pleural effusion. Look for signs of a cause like DVT.
Tests: CBC, Urea & Electrolytes, Clotting profile, D- Dimers. ABGs may show
decreased PaO2 and decreased PaCO2.
Imaging: Chest xray usually normal, ECG may be normal or show right ventricular
strain pattern (inverted T in V1 – V4). Check OHCM Pg. 183 for table showing
diagnosis and scoring.
Management:
* Establish IV access.
* Start LMWH – tinzaparin 175 units/kg/24h subcutaneous.
* Start colloid if systolic BP less than 90, manage BP accordingly, prevent
hypovolemia.
* If BP is more than 90, start Warfarin. Loading regime 5-10mg PO.

90
RESPIRATORY-System Wise 1700-by Sush and Team. 2016
Susmita, Asad, Manu, Saima, Zohaib, Savia, Shanu, Mona, Manisha, Sitara, Samreena, Sami and Komal

* If obvious remedial cause, 6 weeks of warfarin is enough.
* If recurrent embolus or underlying malignancy, give Warfarin for more than 3-6
months.
* Investigate underlying cause.
* Stop heparin when INR is >2 and start warfarin.
* Thrombolysis for a massive pulmonary embolism, give alteplase 10mg IV over 1
minute, then 90mg IV over 2 hours. Max. 1.5mg/kg if less than 65 kg. Consider
placement of a vena caval filter in patients who develop emboli despite adequate
anti-coagulation.


1696. A 7yo boy with frequent episodic asthma is on tx with sodium cromoglycate.
His physician wants to add a non-steroid preventer. The mother of the boy, a
teacher, has just read about a nonsteroidal medication which acts on the mast cells,
stopping them from releasing harmful chemicals. Her physician agrees to add this
medication to the boy's drug regimen. Which medication is the physician most likely
to add to the boy's treatment?
a. Inhaled short acting bronchodilator
b. SC adrenaline
c. Nedocromil Sodium
d. Inhaled long acting bronchodilator
e. Inhaled sodium cromoglycate
f. Inhaled steroids
g. Inhaled SABA
h. Oral steroids
i. Nebulised bronchodilators
j. Oral theophylline


key) C
Mast cell stabilizers include cromolyn sodium and nedocromil. Nedocromil is similar
to cromolyn in its effects and is used for prophylaxis (not acutely).

Investigations for asthma
Spirometry:
FEV1:FVC ratio, A low value indicates that you have narrowed airways which are
typical in asthma Therefore, spirometry may be repeated after treatment. An
improvement in the value after treatment with a bronchodilator to open up the
airways is typical of asthma.
PEFR: morning readings are usually lower than evening readings in asthmatics

91

You might also like